You are on page 1of 86

Uterus : Nonneoplastic and neoplastic lesions,

specimen handling and grossing,


immunohistochemistry and updates.
• Dr Tanushri Mukherjee MD DNB Pathology, Trained in Oncopathology,
Affiliate member of RCPath London

CHAPTER 5
Tumours of the uterine corpus

Epithelial tumours and precursors

Mesenchymal tumours

Mixed epithelial and mesenchymal tumours

Miscellaneous tumours

Lymphoid and myeloid tumours

Secondary tumours
Benign Diseases of the Endometrium
• Embryology and Anatomy • Postmenopausal Endometrium
• Congenital Defects . Endometrial Sampling .... Criteria for Adequacy

Fusion Defects of the Mullerian Ducts . Artifacts in Endometrial Biopsy Specimens . .

Atresia of the Mullerian Ducts and Vagina . Contaminants and Other Elements

• Normal Cyclical Endometrium . .Extrauterine Tissues in Endometrial Biopsy


Endometritis -Chlamydia Trachomatis ...Cytomegalovirus ...
Proliferative Phase ... Secretory Phase . Menstrual Herpes Simplex Virus ... Mycoplasma ..Actinomyces ..Fungi
Lower Uterine Segment Endometrium and Parasites ..
Malakoplakia .. Lymphoma-like Lesion ...Endometrial
• Immunohistochemistry of Normal Granulomas ..Ligneous (Pseudomembranous) Endometritis .
Endometrium
Dysfunctional Uterine Bleeding -Estrogen Related
• Hematopoietic Cells within the Endometrium . Dysfunctional Uterine Bleeding, Including Endometrium
associated with anovulatory Cycles,Progesterone-Related
• Gestational Endometrium . Dysfunctional Uterine Bleeding-Luteal Phase Defects .
Endometrium Associated with EctopicPregnancy . Effects of Exogenous Hormonal Agents
and Drugs ...Estrogen-Only Hormone Replacement
Therapy (HRT) .
Endometrial Epithelial Metaplasia (Epithelial Cytoplasmic
Change) Squamous Metaplasia ..Mucinous
Metaplasia .Ciliated (Tubal) Metaplasia .
Clear Cell Metaplasia .Hobnail Cell Metaplasia
Atypical Polypoid
Eosinophilic (Oxyphilic, Oncocytic) Adenomyoma ..Adenofibroma .
Metaplasia
Effects of Intrauterine Device (IUD) .
Papillary Syncytial Metaplasia
Effects of Mirena Coil ..
Arias-Stella Reaction .
Radiation Effects on the Endometrium .
Papillary Proliferation of Endometrium
Effects of Endometrial Ablation
Endometrial Mesenchymal Metaplasia
Effects of Curettage/resection ..
Smooth Muscle Metaplasia .
Asherman’s Syndrome .
Cartilaginous and Osseous Metaplasia .
Postoperative Spindle Cell Nodule ..
Glial Metaplasia . Psammoma Bodies in the Endometrium .
Adipose Metaplasia . Emphysematous Endometritis .
Extramedullary Hematopoiesis Benign Endometrial Stromal Proliferations
Benign Trophoblastic Lesions ..
Endometrial Polyps
Intravascular Endometrium ..
Endometrial Polyp with Atypical Stromal Cells .
Adenomyomatous Polyp Endometrial Autolysis .
Hyperplasia and Carcinoma Arising in

Endometrial Polyp ..
Uterus
• Pear shaped organ which serves as physiologic site of
implantation for fertilized egg and undergoes changes to
support subsequent placental attachment and embryonic /
fetal development

• Mucosal component (endometrium) is hormonally


responsive and undergoes physiologic and morphologic
changes throughout the menstrual cycle

• Anatomic divisions
◦ Uterine corpus: main portion of the uterus
comprising the upper two - thirds, which houses the
endometrial lined cavity
◦ Uterine cervix: lower one - third of uterus, which
attaches to vaginal canal;
◦ Fundus: domed superior portion of uterus located
superior to points of fallopian tube insertion
◦ Cornua: lateral portions of uterine corpus; sites of
fallopian tube insertion
◦ Isthmus (lower uterine segment): inferior portion of
corpus, which connects to the cervix
◦ Development- The endometrium and the
myometrium are of mesodermal origin and are
formed secondary to fusion of the mullerian
(paramesonephric) ducts between the 8th and 9th
postovulatory weeks.
Anatomy and Vascular anatomy
During the prepubertal years, the endometrium remains inactive, and
the cervix continues to comprise the major part of the uterus. In the
reproductive years, the dimensions and weight of a normal uterus Vascular supply that originates from the radial arteries
varies widely according to parity. of the underlying myometrium.
These arteries penetrate the endometrium at regular
In nulliparous women, the uterus measures approximately 8 cm in
length, 5 cm in width at the level of the fundus, and 2.5 cm in
intervals and give rise to the basal arteries, which in
thickness; most weigh between 40 and 100 g. turn divide into horizontal and vertical branches, the
former providing the blood supply to the endometrial
Multigravid uteri are larger with increasing length and weight with basalis and the latter to the overlying functionalis layer.
increasing parity. The internal os, a fibromuscular junction, separates
the muscular uterine corpus from the fibrous uterine cervix. The The endometrial vessels in the functionalis layer are re-
uterine corpus is divided into the fundus, body, and isthmus. ferred to as spiral arteries.
Their development and arboriation near the endometrial
The fundus is that part of the uterus above the orifices of the fallopian
tubes, and the isthmus represents the lower uterine segment. surface and their connections with the subsurface
epithelial precapillary system, as well as extreme
The uterus is located between the rectum (posteriorly) and the urinary coiling during the menstrual cycle, are influenced by
bladder (anteriorly); it is supported by the round ligaments and the ovarian steroid hormones and prostaglandins.
utero-ovarian ligaments and covered by the pelvic perito- neum.
A differentiating feature between the endometrial and
The endometrium during the reproductive period undergoes cyclical myometrial arteries is the absence of subendothelial
morphologic changes, which are particularly evident in the superficial elastic tissue in the endometrial arteries, except for
two thirds, the so-called functionalis layer. those in the basal layer, and its presence in the
Morphologic alterations are minimal in the deeper one third, the so- myometrial arteries.
called basalis layer. Veins and lymphatics are closely associated with the
endometrial arteries and glands, respectively.
In postmenopausal women, the endometrial morphology is similar to
that in the prepubertal years Uterine lymphatics drain from subserosal uterine
plexuses to the pelvic and para-aortic lymph nodes.
Histology
• Histologic divisions
◦ Endometrium: mucosal layer lining the
uterine cavity composed of endometrial
glands and specialized stroma; blood
supplied by spiral arteries
▪ Stratum basalis: deep layer of
endometrium, which is minimally
hormone responsive and serves to
replenish the stratum functionalis
following menses
▪ Stratum functionalis: hormone
responsive superficial layer of
endometrium; undergoes
functional and morphologic
changes throughout menstrual
cycle; shed during menses
◦ Myometrium: structural wall of uterus
composed primarily of smooth muscle
◦ Serosa: thin, outermost layer of uterus
consisting of loose connective tissue
and mesothelium
Positive stains
• Endometrial glands: PAX8, CK7, ER, PR
• Endometrial stroma: CD10, ER, PR, IFITM1
• Myometrium: SMA, desmin, h-caldesmon
Physiology
• In infants and children, the endometrium is functionally inactive
• Menarche:
◦ First occurrence of menstruation
• In gestational aged women, the endometrium undergoes hormonally driven
changes throughout the menstrual cycle
• Menstrual phase (days 0 - 5):
▪ Estrogen and progestin levels fall in the absence of
implantation of a fertilized egg, resulting in breakdown of
endometrial stroma
▪ Stratum functionalis is shed; spiral arteries constrict to minimize
blood loss
◦ Proliferative phase (days 6 - 14):
▪ Stratum functionalis is regenerated by cells from stratum
basalis
▪ Primarily driven by increasing estrogen levels
▪ Corresponds to follicular phase of cycle in ovary
▪ Ends at approximately day 14 when ovulation occurs
◦ Secretory phase (days 15 - 28):
▪ Stratum functionalis undergoes changes to support
implantation in the event of fertilization
▪ Glands become convoluted and endometrial cells increase
glycogen stores
▪ Primarily driven by progestin
▪ Corresponds to luteal phase of cycle in ovary
• Following menopause, the endometrium becomes inactive and may
eventually undergo atrophy
• Pregnancy changes
◦ Endometrium retains secretory phenotype and stroma becomes
decidualized in response to progestins
◦ Myometrium undergoes mechanoadaptation to allow distension and
accommodation of developing fetus .
Congenital Defects -Mullerian duct abnormalities

Congenital abnormalities of the uterus are Abnormalities of fusion


uncommon. They may be secondary to the effects of Normally, the upper one third of the vagina and the uterus are
exogenous hormones, such as diethylstilbestrol formed by fusion of the paired müllerian ducts. After fusion, the
(DES), in utero or imbalances in endogenous intervening wall degenerates, forming the endometrial cavity and
hormones associated with abnormal gonads and the upper vaginal canal.
chromosomal defects. In utero exposure to DES is, Nonfusion of the müllerian ducts results in a bicornuate uterus. If
of course, nowadays rare. the ducts fuse but the wall between the two lumens persists, an
abnormal septate uterus results. Occasionally a carcinoma develops
Genotypically normal females with normal gonads in one cavity, and only the other normal cavity is sampled during
may also have mullerian duct abnormalities. These the investigation of abnormal uterine bleeding.
developmental aber- rations, such as defects in the If the defect is minor or confined to the fundus, the uterus is
fusion of the müllerian ducts, are caused by errors referred to as arcuatus.
in embryogenesis. If the full length of the uterus and the upper vagina is divided by a
septum, the condition results in uterus didelphys with a partially
These disorders are frequently associated with double vagina.
malformations in the urinary system and the distal
These congenital anomalies may result in infertility or spontaneous
gastrointestinal tract. abortion and in some cases require surgical correction.
Atresia of the Mullerian Ducts and Vagina
Atresia of the müllerian ducts and the vagina may be partial or
complete. The etiology of these conditions is obscure, although a
genetic cause is suggested .
In Rokitansky–Kuster–Hauser syndrome, a severe defect
characterized by mullerian and vaginal aplasia, patients may have
urinary tract anomalies such as a pelvic kidney or anephria.
Proliferative phase
• Endometrium
◦ Proliferative phase
▪ Cellular blue appearance at low
power
▪ Round to tubular glands
▪ Even, regular spacing between
glands
▪ Pseudostratified columnar cells in
glands
▪ Numerous mitotic figures in glands
and stroma
◦ Interval phase (Day 16)
▪ Partially developed subnuclear
vacuoles
▪ Mitoses present, but not as
numerous as in proliferative phase
▪ Less than 50% of cells in a gland
with continuous and well developed
subnuclear vacuoles
Secretory phase
Secretory phase
▪ Relatively pink appearance at low power
▪ Convoluted, irregularly shaped glands
▪ Single layer of columnar or cuboidal cells in glands
▪ Early secretory phase (Day 17 - 19)
▪ Day 17: Continuous and well developed subnuclear
vacuoles in > 50% of a gland, rare mitoses
▪ Day 18: Sub- and supranuclear vacuoles (piano keys) with
nuclei in the center of cell
▪ Day 19: Nuclei at base of cell, supranuclear vacuoles, start
of luminal secretions
▪ Mid secretory phase (Day 20 - 22)
▪ Day 20: Maximal intraluminal secretions, stromal cells with
hyperchromatic nuclei and high N:C ratio
▪ Day 21: Increased stromal edema
▪ Day 22: Peak stromal edema
▪ Late secretory phase (Day 23 - 27)
▪ Day 23: Predecidua surrounds spiral arterioles
▪ Day 24: Predecidua bridges multiple vessels
▪ Day 25: Thin band of predecidua beneath endometrial
surface
▪ Day 26: Thick band of predecidua beneath surface
▪ Day 27: Abundant predecidua expanding downward from
endometrial surface, increased number of stromal
granulocytes
. Fig. 7.2 . Fig. 7.3
Early secretory phase endometrium. Tubular glands exhibit Mid-secretory endometrium. The glands contain

0 7
subnuclear vacuolation

Benign Diseases of the Endometrium


Secretory phase supranuclear vacuoles with secretions within glandular
lumina

assumed that ovulation has occurred when there are sub-


nuclear vacuoles in at least 50% of the cells in at least 50%
of the glands; scattered subnuclear vacuoles are not reli-
able evidence of ovulation and, as stated, may be seen in
late-proliferative endometrium. In the early secretory
phase, the stroma is indistinguishable from that of late-
proliferative endometrium.
Between the 19th and 23rd day of a typical 28-day cycle
(the mid-secretory phase), the degree of glandular secre-
tion increases. Cytoplasmic vacuoles become supranuclear
and secretions are seen within glandular lumina
(> Fig. 7.3); it is important to realize that secretory mate-
rial within glandular lumina is not specific to secretory
endometrium but may also be seen in proliferative, hyper-
plastic, and malignant endometria. Mid-secretory glands
. Fig. 7.2 are usually angular . Fig. in
7.3 shape, and mitotic activity is no . Fig. 7.4
longer
Early secretory phase endometrium. Tubular glandsapparent. The glandsendometrium.
exhibit Mid-secretory in the superficial layers of the
The glands contain Mid-secretory endometrium. There is marked stromal
subnuclear vacuolation functionalis tendsupranuclear
to exhibit less secretory
vacuoles with secretionsactivity, and, as
within glandular edema
a result, superficial luminabiopsies may produce a false impres-
sion of poorly developed secretory activity. Stromal edema
progressively increases, is most obvious in days 22 and 23, (> Fig. 7.5). The predecidual change results in formation
assumed that ovulation has occurred when andthere are sub-
is most prominent in the mid-zone (> Fig. 7.4). Spiral of the so-called compact layer (stratum compactum) be-
nuclear vacuoles in at least 50% of the cells in at least 50%
arteries become apparent. At this stage, the stromal cells neath the surface epithelium, the deeper layers of stroma
of the glands; scattered subnuclear vacuoles
haveare not reli-
more conspicuous eosinophilic cytoplasm; these cells exhibiting less predecidual change. Sometimes, the
several morphological alterations of the endometrium Because of the stromal collapse, the endometrial glands
are characteristic of DUB, the most common being become disrupted and crowded (> Fig. 7.41). This
se associated with anovulatory cycles or luteal phase glandular crowding may mimic hyperplasia or even
cts. These can be regarded as estrogen-related and

Late secretory spiral arteriole


gesterone-related, respectively, and are discussed in
next sections. Often DUB is managed by hormonal
apy and a biopsy only performed when symptoms
. Table 7.1
Features of endometrial breakdown

ist; the hormone therapy may result in modification Glandular changes

Menstrual phase
the morphology. It is stressed that DUB is not
athological diagnosis but rather a clinical term.
Nuclear (apoptotic) debris in basal cytoplasm of glandular
cells
▪ Endometrial
ommon, but not invariable, feature of biopsies from
ents with DUB is the presence of glandular and stro-
Papillary syncytial metaplasia

stromal
breakdown. The features associated with this are not
Glandular crowding
Stromal changes
que to DUB and are seen in menstrual endometrium
breakdown: dense
in bleeding associated with a variety of organic disor- Stromal collapse

round aggregates
. It is important to recognize the features of break-
wn and to distinguish them from other pathological
Aggregates of stromal cells
Nuclear (apoptotic) debris in stroma
of stromal cells
ons. It is also important to realize that glandular and
mal breakdown is a nonspecific feature and that the
Fibrin thrombi

admixed with
ct endometrium must be assessed to evaluate the
Hemosiderin pigment deposition
Foam cell accumulation
erlying abnormality. Glandular and stromal break-
inflammatory cells
wn may also occur in an atrophic endometrium. The Fibrosis and hyalinization

and blood
nges associated with glandular and stromal breakdown
described in the next paragraphs. In menstrual endo-
▪ Papillary syncytial
rium, the features of breakdown are diffuse and occur
a background of secretory endometrium. In contrast,
metaplasia is
DUB, the background endometrium is typically
secretory in type and breakdown is usually a focal
common, thought
nomenon, resulting in a heterogeneous pattern with
ct fragments of endometrium admixed with fragments
to be a reparative
ibiting the features of breakdown. Furthermore, in

response
nstrual endometrium the changes are acute and there
no features of chronic bleeding, such as hemosiderin
osition and accumulation of foam cells.
The morphological features associated with glandular
stromal breakdown are summarized in > Table 7.1.
early feature is the accumulation of nuclear (apopto-
debris in the basal cytoplasm of the glandular cells
. Fig. 7.40 . Fig. 7.42
Breaking down endometrium. With breakdown, the stromal Breaking down endometrium. With breakdown, fibrin
cells aggregate into stromal ‘‘blue balls’’ thrombi are typically seen within blood vessels

. Fig. 7.41 . Fig. 7.43


Breaking down endometrium. With breakdown, the Chronic breakdown. With chronic breakdown, hemosiderin
endometrial glands become disrupted and crowded pigment may accumulate within histiocytes within the
because of stromal collapse endometrial stroma
Lower Uterine Segment Endometrium
E
Lower uterine segment or isthmic endometrium is o
poorly responsive to steroid hormones, and the p
morphology does not alter significantly during the
t
menstrual cycle; as is the case with the endometrial
l
basalis, lower uterine segment endometrium is not
s
useful for dating the menstrual cycle.
t
Lower uterine segment endometrium is composed of o
inactive poorly developed glands that are often ciliated l
L
They are irregularly distributed, and some may be a
dilated. The stroma typically has a fibrous appear- a
ance, and the stromal cells are more elongate and a
‘‘fibroblast-like’’ than in the corpus, t
. Fig. 7.11
e
Lower uterine segment endometrium may be mistaken Lower uterine segment endometrium. This is composed
a
for a polyp in a biopsy specimen. of a mixture of ciliated and mucinous glands within
h
a fibrous stroma
In the inferior part of the lower uterine segment, the a
glands merge with mucinous type glands from the u
upper endocervix, and the stroma becomes even more ovaries; the endometrium is thus a highly sensitive indi- P
cator of the hypothalamic–pituitary–ovarian axis. Steroid i
fibrous.
hormone control of endometrial epithelial and stromal c
cells is mediated by estrogen receptors (ER) and proges- v
terone receptors (PR). These steroid receptors are proteins t
concentrated in the nuclei of endometrial epithelial and o
pattern, hypersecretory pattern, regenerative pattern, and
monstrous cell pattern [4]. However, there is considerable
overlap between these patterns, and there is no value in
attemptingto subclassify Arias-Stella effect endometrium.
TheArias-Stella reactionmayalsobeseenintheglandular

Arias stella epithelium of the cervix and fallopian tube and involving
endometriosis or vaginal adenosis [109].
Apart from the Arias-Stella reaction, the endometrial
glands may undergo other changes in the presence of
trophoblastic tissue. These include abundant clear glyco-
gen-rich cytoplasm; this overlaps with the Arias-Stella
reaction, but the nuclear enlargement of the latter is not
◦ Gestational changes present.Anotherpregnancy-relatedchangeisthepresence
ofopticallyclearnucleiwithintheendometrialepithelium
▪ Decidual change: [98] (> Fig. 7.17). This may occur in association with the
stroma gains abundant Arias-Stella reaction or independently. This appearance is
. Fig. 7.15
eosinophilic cytoplasm, Gestational endometrium. The stroma is expanded and
due to the intranuclear accumulation of biotin and may
simulate the ground glass nuclei of herpes simplex virus
appears polygonal with composed of decidualized cells with abundant eosinophilic infection [157]. However, the nuclei lack the Cowdry
distinct cell borders cytoplasm type A eosinophilic intranuclear inclusions and nuclear

▪ Arias-Stella reaction in
glandular cells
▪ Nuclear
enlargement and
hyperchromasia
▪ Abundant
eosinophilic
vacuolated
cytoplasm
▪ Hobnail
appearance with
cells protruding
. Fig. 7.16 . Fig. 7.17
into glandular Arias-Stella reaction in pregnancy. There is cellular Optically clear nuclei in pregnancy. Endometrial glands
lumen stratification, vacuolated cytoplasm, and enlargement of in pregnancy may contain cells with optically clear
the epithelial cell nuclei nuclei
Post menopausal/Atrophy
Atrophy
▪ Common in postmenopausal women
due to estrogen withdrawal
▪ Glands composed of inactive low
columnar to cuboidal cells
▪ Glands often detached from stroma,
forming hairpin structures
▪ May have cystic change
7
318 Benign Diseases of the Endometrium

. Fig. 7.19 . Fig. 7.21


Atrophic endometrium. There are widely spaced small Lymphoid aggregates in endometrium. Lymphoid
atrophic tubules aggregates are a normal phenomenon within the
endometrial stroma

usually scanty. High-power examination is required to


confirm an absence of mitotic activity, and this is espe-
cially so in the distinction between proliferative endome-
trium and atrophy with small tubular glands and between
cystic forms of atrophy and simple hyperplasia. The stroma
in postmenopausal endometria may be densely cellular and
composed of ovoid to spindle-shaped cells with scant cyto-
endometrium. The term persistent proliferative endome- more budding and branching. However, there is signifi-
trium has also been used. The underlying causes are cant interobserver variability in the distinction between
complex but many cases may be a result of hypothalamic disordered proliferative endometrium and simple hyper-
Estrogen Related Dysfunctional Uterine Bleeding, including Endometrium
dysfunction. The developing follicles persist for a variable plasia, and, as stated, these form part of a continuum
period of time and produce estradiol before undergoing without sharply defined borders. Disordered proliferative
Associated with Anovulatory Cycles
atresia at which time estrogen withdrawal bleeding occurs. endometrium may occasionally be confused with a polyp
In other cases, estrogen breakthrough bleeding occurs because of the glandular architectural distortion and dila-
when the persisting follicles produce estradiol resulting tation; however, the fibrous stroma and thick-walled stro-
in the proliferating endometrium becoming thicker and mal blood vessels characteristic of a polyp are absent and
outgrowing its blood supply. The usual presentation is disordered proliferation involves the entire endometrium.
with perimenopausal bleeding, but younger women may
also be affected, for example, perimenarchal adolescents in
This endometrial morphology is most
whom regular common
ovulatory cycles in the
are not established, those
with polycystic ovarian syndrome (Stein–Leventhal syn-
perimenopausal years where it orisolderusually
drome), women takingsecondary
unopposed estrogens to or
anovulatory cycles with resultant
with absence
an increase inof development
endogenous ofexample,
estrogens, for the
secondary to obesity. Anovulatory cycles may also occur
corpus luteum and decrease insporadically
progesterone secretion; referred
throughout the reproductive years. The histo-
to as anovulatory or persistent proliferative endometrium.
logical features involve the entire endometrial compart-
ment and are those of proliferative endometrium but with
superimposed
The underlying causes are complex breakdown,
but manyas cases describedmay
above [103].
be aThe
extent of breakdown is highly variable and ranges from
result of hypothalamic dysfunction.
minute focalIn other
changes to acases,
widespreadestrogen
phenomenon
breakthrough bleeding occurs when the persisting follicles
involving most of the specimen. There are often foci of
ciliated (tubal) or other types of epithelial metaplasia that
produce estradiol resulting inare randomly
the proliferating
dispersed. endometrium
becoming thicker and outgrowingWith its blood supply. cycles, there is abundant
chronic anovulatory
proliferative endometrium and mild degrees of disorgani- . Fig. 7.44
zation with dilated glands may occur. This results in Disordered proliferative endometrium. Occasional
The usual presentation is with perimenopausal bleeding, but
a picture, which is neither normal proliferative nor hyper- cystically dilated glands are present within an otherwise
younger women may alsoplasticbeand which affected,
is referred tofor example,
as disordered proliferative typical proliferative endometrium
perimenarchal adolescents in whom regular ovulatory cycles are
not established, those with polycystic ovarian syndrome (Stein–
Leventhal syndrome), or older women taking unopposed
estrogens or with an increase in endogenous estrogens, for
example, secondary to obesity.
are usually, but not always, low grade and early stage.
Unopposed estrogens result in endometrial hyperplasia
in approximately 20% of women following 1 year of treat-
ment. The ‘‘postmenopausal estrogen/progestin interven-

Progesterone-Related Dysfunctional Uterine tion (PEPI) trial ’’ concluded that women taking estrogens
alone had a high incidence of simple (27.7%), complex
(22.7%), and atypical (11.7%) hyperplasia; this was sig-

Bleeding-Luteal Phase Defects nificantly higher than in those taking placebos. The
reported risk ratio for endometrial carcinoma in women
taking unopposed estrogens has ranged from 2.3 to 10 [53,
116]; the risk persists for many years after estrogen treat-
ment is discontinued [15, 132]. Estrogen-only prepara-
tions may also result in proliferative changes and the
development of premalignant and malignant lesions in
endometriosis; as such, caution should be exercised before
. Fig. 7.45
prescribing unopposed estrogens following hysterectomy
Endometrium associated with continuous combined
in a woman with known endometriosis.
hormone regimes. With continuous combined hormone
regimes, the endometrium is usually atrophic
Luteal phase defects (also known as inadequate
Combined luteal phase, and Progestin Hormone
Estrogen
Replacement Therapy age group, there is a background incidence of endometrial
secretory insufficiency, or inadequate secretory phase) are a Effects of Exogenous Hormonal Agents and
hyperplasia and carcinoma. With the continuous com-
Because of the potential adverse effects of unopposed bined regimes, the endometrium is usually atrophic or
relatively common cause of DUB and also of ovulatory exhibits weak secretory activity ( Fig. 7.45) and, in >
estrogens,

infertility.,insufficiency of progesterone combined Drugs


in most women with a uterus an estrogen is
with a progestin for HRT. Estrogen and proges-
tin combinations may be given sequentially or continu-
many instances, biopsies yield scanty material. There is
no increased risk of endometrial proliferative lesions with
ously. Sequential (cyclic) regimes are variable but usually continuous combined HRT [105], and, in fact, these
employ daily estrogens for the first 21 to 25 days or the regimes may protect against the development of endome-
As a consequence, the luteal (secretory) phase does not Endometrial morphology does not correspond to a normal
whole of the month with daily progestins added for the last trial hyperplasia and carcinoma and normalize endome-
10 to 13 days; these regimes result in a withdrawal bleed. tria that have exhibited complex hyperplasia [40, 43, 136].
develop appropriately, the secretory features in the cyclical or postmenopausal appearance.
Continuous combined regimes use both estrogen and This suggests that continuous combined HRT is suitable
progestin daily. With continuous regimes, breakthrough for long-term use in perimenopausal and postmenopausal
endometrium being poorly developed. bleeding may occur during the first 6 months, but this women. Endometrial polyps are relatively common in
bleeding then usually stops. Patients receiving sequential women taking combined HRT and appear more common
Estrogen-Only Hormone Replacement Therapy
or continuous regimes may undergo biopsy as part of
Hypothalamic or pituitary dysfunction, which results in
routine surveillance or when unexpected bleeding occurs;
with sequential than continuous regimes [43].

decreased levels of follicle stimulating hormone (FSH) and (HRT)


there is no correlation between bleeding patterns and
endometrial histology. With sequential regimes, the endo- Progestin-Only Compounds
metrium may exhibit atrophy, secretory, or weak prolifer-
abnormal luteinizing hormone (LH) secretion. Because of
ative activity, the latter especially if the biopsy is taken Various forms of synthetic analogues of progesterone,

inadequate progesterone secretion, there may be a lag in the Estrogen-only HRT (unopposed estrogen) is contraindicated
during the period of estrogen therapy. If the endometrium
is biopsied during the period of progestin therapy, there
termed progestins, are in widespread use, either alone or
in combination with an estrogen. Progestin-only hor-

7
monal compounds, taken either orally or systemically,
histological date of the endometrium of at least two days due to the risk of endometrial proliferative lesions,
may be poorly developed secretory activity in the glands
with cytoplasmic vacuoles and scant luminal secretions. are usually prescribed for abnormal uterine bleeding and
result in suppression of ovulation and inhibition of endo-
compared to the actual postovulatory date. 334 including hyperplasia and endometrioid adenocarcinoma,
Focal glandular and stromal breakdown may also be seen.
Benign Diseases of the Endometrium
Sequential regimes do not completely eliminate the risk of metrial growth. Progestins may also be given for the man-
carcinoma associated with unopposed estrogen therapy; agement of conditions such as endometriosis, for
the prevalence of endometrial hyperplasia associated with contraception, or for endometrial protection in patients
Other morphological features in some cases include
sequential HRT is 5.4% and that of atypical hyperplasia
0.7% [140]. It should be remembered that HRT is most
taking tamoxifen. The effects of progestins on the endo-
metrium are variable and depend on the degree of estro-
discordance in development of the glands and stroma and
commonly taken by postmenopausal women, and in this gen priming as well as the type of progestin and the dose

different areas of the endometrium exhibiting marked


variation in development; for example, some areas may
exhibit early secretory activity while others show predecidual
change.
Alternatively, the glands may exhibit hypersecretory features
while the stroma lacks predecidual change.

. Fig. 7.46 . Fig. 7.47


Endometrium associated with progestin-only compound. Decidualized endometrial stroma. Occasionally
With progestin-only compounds, the stroma is typically decidualized stroma contains cells with enlarged nuclei
expanded and composed of decidualized cells with cytoplasmic vacuolation and stromal myxoid change
carcinoma (serous EIC). In such instances, immunohisto- spindle shaped, evenly spaced nuclei, sometimes with
chemistry may be of value in that serous EIC and type 2 small nucleoli. Some of these nuclei may contain optically
cancers often exhibit diffuse intense nuclear p53 immu- clear biotin-rich inclusions. The cell borders are indis-
noreactivity while ER is generally negative or weakly pos- tinct. Mitoses are rare or absent. There may be central
itive. In contrast, most epithelial metaplasias are ER necrosis. It is controversial whether morules actually
positive and exhibit a pattern of p53 immunoreactivity, exhibit squamous differentiation. Morphological features
which has been described as weak and heterogenous [123]. of overt squamous differentiation, such as keratinization,
A minor population of endometrial epithelial cells exhibit intercellular bridges, and prominent cell membranes, are

Metaplasia
nuclear immunoreactivity with p63; it has been speculated typically absent in morules. Immunohistochemically,
that these are reserve cells or basal cells and the origin of
the various epithelial metaplasias [111].

Squamous Metaplasia

Squamous metaplasia is one of the commonest forms of


endometrial epithelial metaplasia. Although usually
a focal finding, on occasions there may be widespread

Squamous metaplasia is one of the commonest forms squamous metaplasia with obliteration of the glandular
lumina such that it is difficult to assess the underlying
glandular component. This is especially common when
of endometrial epithelial metaplasia. Although usually
the squamous metaplasia is of morular type (see below).
Squamous metaplasia is common in endometrioid ade-

a focal finding, on occasions there may be widespreadnocarcinoma and in endometrial hyperplasias; these
should be excluded by careful examination of the glandu-
lar elements. Squamous metaplasia may also be seen in . Fig. 7.48
squamous metaplasia with obliteration of the glandular
endometrial polyps. There are two types of squamous
metaplasia, namely typical squamous metaplasia and
Squamous metaplasia in endometrium. Typical squamous
metaplasia with obvious squamous differentiation in the

lumina such that it is difficult to assess the underlying


morular metaplasia, although these sometimes coexist. form of prominent cell membranes

338 7 Benign Diseases of the Endometrium

glandular component. This is especially common when


the squamous metaplasia is of morular type
Squamous metaplasia is common in endometrioid
adenocarcinoma and in endometrial hyperplasia, Benign Diseases of the Endometrium 7 339

endometrial polyps. There are two types of squamous


metaplasia, namely typical squamous metaplasia and . Fig. 7.49
Squamous morules in endometrium. Squamous morules are
. Fig. 7.51
CDX2 immunohistochemistry in squamous morules.
composed of rounded aggregates or syncytial sheets of Morules exhibit nuclear immunoreactivity with the

morular metaplasia, although these sometimes coexist. cells filling glandular lumina intestinal transcription factor CDX2

exhibit nuclear immunoreactivity with the intestinal tran-

Typical squamous elements are characterized by sheets scription factor CDX2 (> Fig. 7.51) [62, 152]; it has been
suggested that this is secondary to beta-catenin gene
mutation. In contrast, typical squamous elements are usu-

of cells exhibiting obvious squamous differentiation in ally positive with ER, p63, and CD10, and negative with
CDX2. On the basis of the immunophenotype, it has been
concluded that morules exhibit no firm immunohisto-

the form of intercellular bridges, prominent cell chemical evidence of squamous differentiation, although
immature squamous features cannot be excluded [62]. It
has been suggested that the term morular metaplasia is
membranes, or keratinization. Sometimes there is a . Fig. 7.52
used instead of squamous morules.
. Fig. 7.53

histiocytic and giant cell reaction to keratin. Mucinous metaplasia in endometrium. Focally the cells
have abundant mucinous cytoplasm
Mucinous Metaplasia
Ciliated metaplasia in endometrium. The endometrial
glands are lined by ciliated cells with abundant eosinophilic
. Fig. 7.50 Mucinous metaplasia is a relatively uncommon form of
Beta-catenin immunohistochemistry in squamous morules. cytoplasm epithelial metaplasia and is most commonly
endometrial
seen in association with a premalignant or malignant
Mucinous and Ciliated Metaplasia Morules exhibit nuclear and cytoplasmic immunoreactivity
intestinal metaplasia [80]. In mucinous metaplasia with-
with beta-catenin
out an associated premalignant or malignant glandular
lesion. It may also be seen in endometrial polyps.
Amade only when
diagnosis one or more
of mucinous endometrial
metaplasia glands
should becontain
reserved
for cases in which the endometrial epithelial cells are
proliferation, there are often small micropapillary projec- ciliated cells, which may be interspersed among non-
morules exhibit a somewhat different immunophenotype replaced by cells with abundant intracytoplasmic mucin,
to tions. Thetypical
that of nuclei squamous
are small and uniformMorules
elements. and mitoses are
exhibit ciliated
the cells cells or be extensive
resembling and line
endocervical cellsmost of 7.52).
(> Fig. the gland
Nor-
rare orand
nuclear absent. An important
cytoplasmic point with
positivity a florid
with mucinous
beta-catenin (> Fig.endometrial
mal 7.53). The nuclei may be cells
epithelial rounded, mildly astrati-
contain little
(>proliferation
Fig. 7.50) [14,
of124] while in typical
the endometrium is squamous elements
that mucinous ade- intracytoplasmic
fied, and containmucin, small especially withare
nucleoli but a luminal distri-
cytologically
the ‘‘normal’’ membranous pattern of immunoreactivity bution, and so abundant intracytoplasmic mucin is
nocarcinomas, even those exhibiting myometrial invasion,
is maintained [62]. Endometrial proliferative lesions with
bland. Ciliated
required cells often
to diagnose have metaplasia.
mucinous abundant eosinophilic
Rarely, intes-
can be often
morules cytologically
exhibitbland with littlegene
beta-catenin in themutation,
way of mitotic
and cytoplasm.
tinal metaplasiaCiliated
has metaplasia is particularly
been described associated
in the endometrium
this resultsAs
activity. in such,
the cytoplasmic and nuclear
complex mucinous immunoreac-
proliferations of the where the mucinous epithelium contains
with estrogenic stimulation. As with other types goblet cells
of epi-
Eosinophilic or oxyphilic metaplasia is relatively common
tainfeatures
and is characterized
small
byand
of glandular
breakdown such
theadjacent
presence
as apoptotic
lumina
of epithelial
anddebris,
which
cells with
neutro-
are devoid of Although there is nuclear enlargement an
phils, glandular and stromal breakdown.
stromal cytoplasm
abundant eosinophilic support, (> lacking
Fig. 7.56).fibrovascular
The cyto- stromal cores nuclear to cytoplasmic ratio is maintained
plasm may be Fig. 7.10).
(>granular, in whichThecase cells
the usually have eosinophilic cyto-
term oncocytic
plasm
metaplasia has beenand there
used. is often a neutrophilic
Ultrastructurally, abundant infiltrate. There
cytoplasmic may
mitochondria
be mildmay be present,
nuclear as is and
atypia in a minority of cases, Papillary Proliferation of Endome
character-
istic of oncocytes
mitoses in other organs. The
are present. Theterm pink cell between papillary
distinction
metaplasia has also been used. Ciliated metaplasia
syncytial metaplasia and serous adenocarcinoma is or serous The term hyperplastic papillary proliferati
often characterized by abundant eosinophilic cytoplasm
EIC has been discussed earlier. Another important con- metrium has been used for a lesion, usual
and overlaps with eosinophilic metaplasia. The epithelial
sideration is that foci similar to papillary syncytial meta- postmenopausal women, characterized by
cells in eosinophilic metaplasia can exhibit a significant
plasiaatypia
degree of nuclear may(>occur on this
Fig. 7.57); the issurface
analogous oftosome endometrioid papillae with fibrovascular stromal core
adenocarcinomas.
the degenerative nuclear atypia The thatdistinction
is commonbetween in papillary syn- degrees of branching and cellular tufting
cytial
oncocytic cells metaplasia
in other organs. The main and differential
papillarydiag- adenocarcinomas of The papillae are lined by epithelial cells wi
nosis is the endometrioid
eosinophilic
. Fig. 7.57 or
or serous type
oxyphilic is facilitated
variant of by recognition
. Fig. 7.58 [76]. Although not strictly a metaplasia
. Fig. 7.55
endometrioid adenocarcinoma.
Eosinophilic Distinction
metaplasia in from
endometrium. adeno-
. Fig. 7.56 that papillary syncytial metaplasia is limited to the endo-
In eosinophilic Papillary discussed
proliferation here Papillary
of endometrium. since epithelial metaplasi
carcinoma is metaplasia,
based on thea significant
absence degree
of a of nuclear
grossly atypia may be
visible projections lined by bland epithelial cells on the surface of
Hobnail.cell
Fig.metaplasia
7.54 in endometrium. Hobnail cells are metrial
Eosinophilic metaplasia surface
present in and
endometrium. is associated
The with
epithelial other morphological monly
an endometrial polyp
mucinous, eosinophilic, or ciliated
lesion and features
maintenance of the normal such asglandular
present Clear
on the cellsurface
metaplasia
of aninendometrial
endometrium.polyp
The endometrial cells contain abundant of breakdown
eosinophilic cytoplasm apoptotic debris, neutro-
glands are replaced by cells with abundant clear cytoplasm architecture.
phils, and adjacent glandular and stromal breakdown.

. Fig. 7.57 . Fig. 7.58


. Fig. 7.55 . Fig. 7.56 Eosinophilic metaplasia in endometrium. In eosinophilic Papillary proliferation of endometrium. Pap
Hobnail cell metaplasia in endometrium. Hobnail cells are Eosinophilic metaplasia,
metaplasia in endometrium.
a significant The epithelial
degree of nuclear atypia may be projections lined by bland epithelial cells o
present on the surface of an endometrial polyp cells contain present
abundant eosinophilic cytoplasm an endometrial polyp
Polyps
Benign Endometrial Polyp
Localized, disorganized altered glands and stroma.
Glands: tubules that may be simple, branched, or
cystically dilated. Lined by inactive epithelium.
Stroma: often collagen-rich containing
characteristic thick blood vessels
Often solitary. Can be anywhere in uterus.
Small polyps are often asymptomatic.
Large polyps may cause bleeding.
May have superimposed metaplasia, hyperplasia, or
carcinoma (particularly SEIC in postmenopausal)

Atypical Polypoid Adenomyoma “APA”


Three key features:
1) Endometrial glands with some architectural
complexity and cytologic atypia 2
2) Prominent squamous morules
3) Surrounding prominent cellular fibromuscular stroma 1
Often centered in lower uterine segment. ~2cm.
Associated with MLH-1 promoter methylation (~1/2)
Can be confused with myoinvasive endometrioid
adenocarcinoma, but APA fibromuscular stroma is P16+
(whereas desmoplastic stroma/myometrium is P16-)
Although benign, can progress or be associated with 3
atypical hyperplasia/EIN or endometrioid
adenocarcinoma

Adenosarcoma (Think: Phyllodes tumor)


Mixed epithelial and mesenchymal tumor with a benign
epithelial component and low-grade malignant stroma.
Papillary/polypoid projections of cellular stroma into dilated
gland l men Of en i h conden a ion colla ing a o nd
benign surface glands. Stroma resembles endometrial stroma
but is often more fibroblastic.
Often post-menopausal but can be any age.
Can show heterologous elements and sarcomatous
overgrowth. When of mo i a high-grade sarcoma,
Adeno a coma i h a coma o o e g o h
IHC: Stroma (+) CD10, ER, PR.
Prognosis: Recurring potential. If sarcomatous overgrowth,
more aggressive can metastasize.
edly atypical symplastic-like nuclei ( Fig. 7.64), resem-
may be large. There are no pathognomonic histological
bling those seen in polypoid lesions elsewhere in the
postmenopausal and the surrounding endometrium is features ofnuclei
appearing tamoxifen-associated
(> Fig. 7.62), andendometrial
sometimes hobnailpolyps,
female genital tract, such as in fibroepithelial stromal
atrophic. The presence of proliferative activity in a polyp but change.
cell an increased
There mayincidence of surface
be focal epithelialglandular
metaplasias,
and polyps of the vulva and vagina [143]. These atypical stro-
in a postmenopausal woman is of no clinical importance, periglandular
stromal stromal
breakdown and,condensation,
on occasions, stromal
polypsedema, and
are exten- mal cells are of no significance.
although it is useful to comment in the pathology report on myxoid
sively changesecondary
necrotic and staghorn-shaped
to torsion or glands
if they polarized
outgrow
whether non-polypoid endometrium is also present and alongblood
their the long axis of
supply; the polyp
vascular have beenmay
thrombosis reported
be seen [72].
in
suchAscases.
stated, diagnosing an endometrial polyp
in isthe
gener- Adenomyomatous Polyp

Endometrial Polyp
whether this exhibits proliferative activity. The stroma of In some instances, this may result for-
a polyp is often more fibrous than that of the non-polypoid ally straightforward
mation of a necrotic when the polyp
polypoid is large
mass with onlyand
theremoved
surface
endometrium but this is not invariable and, in some polyps, intact. However,
epithelium or thewhen small
ghost and fragmented,
outlines of glands the diagno-
remaining As discussed, some endometrial polyps contain a minor
the stroma is dense and cellular and resembles that of sis is more difficult. Lower uterine segment
(> Fig. 7.63). Variable amounts of stromal edema and endometrium component of stromal smooth muscle bundles, often in
normal proliferative endometrium. As stated, collections may be mistaken
occasionally myxoid forchange
a polypmaybecause of theasirregular
be present well as close proximity to thick-walled blood vessels. When the
of thick-walled stromal blood vessels are a characteristic glandular architecture and fibrous stroma. The spindle
feature of endometrial polyps, and ectatic thin-walled ves- cell alteration of the stroma seen in some cases of endo-
sels are also sometimes seen. Some authors have divided metritis may resemble the fibrous stroma of a polyp. How-
endometrial polyps into different types, such as prolifera- ever, other morphological features of a polyp are absent
tive/hyperplastic (proliferative, sometimes crowded, and there is a plasma cell infiltrate within the stroma; it
The histological features of a polyp, not all of which are present in
glands), atrophic (atrophic glands), and functional (glands should be remembered, however, that plasma cells may
occur within the stroma of an endometrial polyp. Espe-
resembling those in the surrounding cyclical endome-
every case, include the following: trium). However, these patterns often overlap and assign-
ment to a specific type may be difficult; moreover, there is
cially in large polyps with a degree of stromal condensa-
tion and increased cellularity around the glands, the
no clinical significance attached to the different types. Some differential diagnosis may include an adenosarcoma.
polyps originate at the junction of the upper endocervix and Adenosarcoma typically has a leaf-like or club-like archi-
• 1  Polypoid pieces of tissue lined by epithelium on three sides,
lower uterine segment and contain both endocervical and tecture, with broad papillae lined by surface epithelium,
and intraglandular stromal projections, the overall archi-
ciliated lower uterine segment type glands.
A variety of morphological appearances affecting the tecture resembling a phyllodes tumor of the breast. In
epithelium or stroma may be seen in endometrial polyps contrast, endometrial polyps usually have a smooth out-
• 2  Glands set in a stroma that is qualitatively different than the
and can result in diagnostic difficulty. Papillary prolifera- line. The stroma in adenosarcoma is usually more cellular
tions with fibrovascular cores (see section > Papillary than in a benign polyp with increased mitotic activity and
endometrial stroma in the non-polypoid frag- ments. The Proliferation of Endometrium) occasionally occur on
.
a Fig.
degree7.62of nuclear atypia, especially immediately sur-
Endometrial
rounding thepolyp. TheWith
glands. epithelium
multipleonrecurrent
the surface of a polyp
endometrial
. Fig. 7.64
Endometrial polyp with atypical stromal cells. Rare
the surface of an endometrial polyp or within cystically
stroma is often, but not always, more fibrous than that in the
dilated glands. The epithelium on the surface of a polyp
may exhibit a degree of atypia, often with degenerate
may
polyps,exhibit
appearance
a degreeofofadenosarcoma
a diagnosis nuclear atypiashould
since the morphological features may be subtle.
with a degenerate
be suspected endometrial polyps contain stromal cells with markedly
atypical nuclei
Benign Diseases of the Endometrium 343 7
non-polypoid fragments and is sometimes markedly hyalinized thought to be related in some way to hyperestrogenism,
possibly originating as a localized hyperplasia of the endo-
metrial basalis secondary to hormonal influences. There is

• 3  Glandular architectural abnormality with dilated glands and an increased incidence of endometrial polyps with HRT
usage, either estrogen-only HRT or combined prepara-
tions. Tamoxifen is also associated with an increased risk
sometimes mild glandular crowding of the development of endometrial polyps. Molecular
studies have demonstrated that many endometrial polyps
represent monoclonal endometrial stromal overgrowths
with secondary induction of polyclonal benign glands

4 Glands that appear different to those in the surround- ing through undefined stromal–epithelial interactions [46].
They may contain abnormalities of chromosome 6 [34].
Polyps may be single or multiple, sessile or broad
endometrium; for example, the glands of the non- polypoid based, pedunculated or attached to the endometrium by
a slender stalk. They usually have a smooth surface and
. Fig. 7.59

endometrium may be secretory in type while the glands within the small cysts may be seen on sectioning. They can arise
anywhere in the endometrium, including the lower uter-
Endometrial polyp. Dilated glands are set in a fibrous
stroma
ine segment, but are most common in the fundus. When
polyp are atrophic or exhibit poorly developed
. Fig. 7.61 secretory or
Endometrial polyp. Collections of thick-walled stromal
. Fig. 7.63
large, they may fill the endometrial cavity and extend into
Necroticthe endocervicalpolyp.
endometrial canal. Necrosis has occurred

proliferative activity, blood vessels are a characteristic feature of endometrial


polyps
The pathological diagnosis is generally straightforward
secondary to torsion
if the gynecologist is aware of the presence of a polyp, has
conveyed this information to the pathologist, and has
removed the polyp intact. On occasions, the gynecologist
believes that a polyp is present but histological examina-
5 Collections of thick-walled stromal blood vessels tion shows a cyclical endometrium, often secretory in type,
reflecting the fact that an abundant secretory endome-
trium may have a polypoid appearance. In many cases,
the gynecologist is not aware of the presence of a polyp,

The glands within a polyp are usually endometrioid in type but not which is removed piecemeal with the result that in biopsy
material, fragments derived from the polyp are admixed
with fragments of non-polypoid endometrium, making
uncommonly exhibit metaplastic change, including ciliated, the diagnosis difficult. In biopsies performed because of
abnormal uterine bleeding, the pathologist should always

eosinophilic, mucinous, and squamous metaplasia. The epithelium consider the possibility of a polyp. Under low-power
examination, the initial clue to the diagnosis is often the
. Fig. 7.60
Endometrial polyp. There may be a mild degree of glandular
admixture of fragments of normal cyclical or atrophic crowding within some endometrial polyps
may be atrophic but often exhibits proliferative activity, endometrium and fragments that are morphologically
different. The histological features of a polyp, not all of 4 Glands that appear different to those in the surround-
which are present in every case, include the following: ing endometrium; for example, the glands of the non-
polypoid endometrium may be secretory in type while
1 Polypoid pieces of tissue lined by epithelium on three
the glands within the polyp are atrophic or exhibit
sides,
poorly developed secretory or proliferative activity,
2 Glands set in a stroma that is qualitatively different
5 Collections of thick-walled stromal blood vessels
than the endometrial stroma in the non-polypoid frag-
(> Fig. 7.61).
ments. The stroma is often, but not always, more
fibrous than that in the non-polypoid fragments and The glands within a polyp are usually endometrioid in
Atypical polypoidal adenomyoma
Atypical Polypoid Adenomyoma Benign Diseases of the Endometrium 7 347

serous proliferation since the cells of EIC typically exhibit


diffuse intense nuclear immunoreactivity with p53

Atypical polypoid adenomyoma is a biphasic polypoid (> Fig. 7.67) and there is a high MIB1 proliferation
index. ER is usually negative. In contrast, the benign
epithelium within the polyp exhibits a low MIB1 prolifer-

lesion composed of endometrioid type glands in a ation index, is ER positive and is negative or only scattered
nuclei are positive with p53. p53 staining may reveal that
the EIC is more extensive than is appreciated on initial

myomatous or fibromyomatous stroma . Since the stroma morphological examination. Rarely a carcinosarcoma
arises in and is confined to an endometrial polyp. Occa-
sional cases of metastatic carcinoma, especially breast
may be fibromyomatous also atypical polypoid lobular carcinoma, have been reported in endometrial
polyps [60].

adenomyofibroma Atypical Polypoid Adenomyoma . Fig. 7.68


Atypical polypoid adenomyoma. Endometrioid type glands
are embedded within a myomatous stroma
Atypical polypoid adenomyoma is a biphasic polypoid
lesion composed of endometrioid type glands in

.Most patients are premenopausal or perimenopausal a myomatous or fibromyomatous stroma [97, 159].
Since the stroma may be fibromyomatous rather than
overtly myomatous, some prefer the designation atypical
(average age 40 years) and present with abnormal uterine polypoid adenomyofibroma [79]. Most patients are
premenopausal or perimenopausal (average age 40 years)
and present with abnormal uterine bleeding, usually in the
bleeding, usually in the form of menorrhagia. form of menorrhagia. In some cases, the diagnosis is made
during investigations for infertility. Occasional cases occur
in postmenopausal women, and rare examples have been
described in patients with Turner’s syndrome who have
been prescribed unopposed estrogens [23]. A single study

In some cases, the diagnosis is made during investigations has investigated molecular events in atypical polypoid
adenomyoma and found MLH-1 promotor
hypermethylation in some cases, a molecular alteration

for infertility. characteristic of some atypical hyperplasias and


endometrioid adenocarcinomas [114].
. Fig. 7.69
Atypical polypoid adenomyoma is most commonly
located in the lower uterine segment, although some Atypical polypoid adenomyoma. There is abundant
cases involve the fundus, uterine body, or endocervix. In squamous morule formation

MLH-1 promotor hypermethylation molecular alteration most cases, the lesion has an obvious polypoid gross
appearance, in the form of either a sessile or broad-based
polyp, but sometimes the polypoid nature is not grossly mild or, at the most, moderate cytological atypia. Occa-
obvious, especially in smaller lesions. sional foci of ciliated or mucinous epithelium may be
The diagnosis may be made on endometrial biopsy, or present. A characteristic histological feature that is present

Atypical polypoid adenomyoma is most commonly located


polypectomy, or at hysterectomy. Histology shows archi- in most, but not all, cases is abundant squamous morule
tecturally irregular endometrioid type glands that may be formation (> Fig. 7.69); sometimes, the morules exhibit
widely separated and haphazardly arranged or somewhat central necrosis. The glands are set in an abundant stroma,

in the lower uterine segment, although some cases involve crowded and arranged in groups, sometimes with
a vaguely lobular pattern (> Fig. 7.68). The endometrioid
epithelium varies in appearance from cuboidal to low
which varies from obviously smooth muscle in nature to
fibromyomatous. Endometrial stroma is not present. The
stromal cells are often arranged in short interlacing fasci-

the fundus, uterine body, or endocervix. In most cases, the columnar to pseudostratified. The nuclei are usually
round, sometimes with prominent nucleoli, and exhibit
cles. Occasional mitotic figures may be identified within
the stroma. The margin between the lesion and the

lesion has an obvious polypoid gross appearance, in the


form of either a sessile or broad-based polyp,
Atypical polypoid adenomyoma proliferation of endometrial glands embedded in a
background of intersecting myometrial fascicles; no cytological atypia
may also stain with p53.

Endometrial Polyp Classification and Treatment


Diagnosis Epithelium Stroma Management
Benign Endometrial Polyp Benign Benign, Fibrous Polypectomy
Polypoid adenomyoma Benign Benign, Muscle Polypectomy
CAH/EIN in a polyp Atypical/Crowded Benign, Fibrous Hysterectomy or
Hormones
Atypical Polypoid Adenomyoma Atypical/Crowded Benign, Muscle Hysterectomy or
(APA) Hormones
Adenosarcoma Benign Malignant Hysterectomy
Carcinoma Malignant Benign Hysterectomy
Carcinosarcoma Malignant Malignant Hysterectomy
Endometrial sampling
For evaluation of infertility the timing of the biopsy is Endometrial polyps may be involved by small
important; carcinomas or by serous endometrial intraepithelial
Ideally, the biopsy should be taken in the mid-secretory carcinoma (serous EIC), and, as a rule of thumb, at
phase between the 7th and 11th postovulatory days. least one block per cm should be taken of large
endometrial polyps.
Endometrial polypectomy may be undertaken, and in
such instances, sampling of the non-polypoid Endometrial chips should be orientated if possible
endometrium should also be undertaken. since poorly orien- tated specimens, especially if
Transcervical resection of the endometrium (TCRE) tangentially sectioned, may result in a histological
may be performed for menorrhagia or multiple polyps or suspicion of adenomyosis.
an endometrial hyperplasia or a low-grade endometrioid Potential for overdiagnosis of myometrial invasion
adenocarcinoma . This procedure produces a specimen
in cases of endometrial hyperplasia or
consisting of multiple chippings, similar to prostatic
chips. Chips should be weighed (this should also be adenocarcinoma; again, this is due to poor
done when biopsy or D and C yields a significant orientation and tangential sectioning.
amount of tissue but may be impractical with very scant Adequate clinical history is important, including
specimens). the age of the patient and the reason for biopsy.
All endometrial specimens should be submitted in their Knowledge of the menopausal status as well as the
entirety for histological examination. date of onset of the last menstrual period (LMP)
The exception is large endometrial polyps where and the length of the menstrual cycle in
representative sampling may be undertaken; premenopausal women should be provided.
glands, which is a clue to the artifactual nature (> Fig. 7.24). endometrium
pseudopapillary areepithelium,
present in which
endometrial biopsy
are generally or curet-
atrophic,
An artifact that is especially common with, but not exclu- tage specimens.
should be examinedSuperficial
carefullymyometrium
under high power is commonly
to look
sive to, outpatient biopsies is the presence of superficial seen,
for especially
proliferative in activity
vigorousandcurettage specimens
nuclear atypia. and in
Crushed
. Fig. 7.24
strips of endometrial epithelium, sometimes accompanied postmenopausal
endometrial glands women with anmay
and stroma atrophic endometrial
be extremely cellularlin-
Glandular
by molding. A common
a little stroma, with artifact in endometrial
a pseudopapillary architecture ing.
andIt is very
can common
cause to see
concern. cervicalcrush
Extensive mucus, oftenisadmixed
artifact more

Artefacts,Contaminants
biopsies(>
is glandular
Fig. 7.25). ‘‘molding.’’ There
This may result in is tearing of the
consideration of a wide with neutrophils,
likely to occur inhistiocytes and atrophic
biopsies from giant cells (> Fig. 7.27),
endometrium
tissue around
range ofthepapillary
glands, lesions,
which isbenign
a clue and
to the artifactual
malignant, which and cervical tissue patients.
in postmenopausal in endometrial
As with thebiopsy specimens.
examination of

20 7 Benign Diseases of the Endometrium


nature occur in the endometrium. Such superficial strips of other
The tissues,tissue
cervical crushed elements
usually should
takes the not
formbe of
viewed
isolation. Problems associated with poorly orientated endo-
metrial resection specimens have already been described.
Another artifact that may be seen in endometrial resection
in of
strips

specimens and which is secondary to cautery is vacuolation


of the endometrial stromal cells, resulting in a signet-ring
appearance (> Fig. 7.26); this is a similar phenomenon to
the vacuolation of cervical stromal cells, which may occur
secondary to cautery [93].

Contaminants and Other Elements in


Endometrial Biopsies

Not uncommonly, fragments of tissue other than from the


endometrium are present in endometrial biopsy or curet-
tage specimens. Superficial myometrium is commonly
seen, especially in vigorous curettage specimens and in
. Fig. 7.24 postmenopausal women with an atrophic endometrial lin-
. Fig. 7.25
Glandular molding. A common artifact in endometrial . ing.
Fig. It7.26
is very common to see cervical mucus, often admixed
Pseudopapillary
biopsies isendometrium. Endometrial
glandular ‘‘molding.’’ There isbiopsy
tearing of the Vacuolation of endometrial
with neutrophils, histiocytesstromal cells.
and giant cellsVacuolation,
(> Fig. 7.27),
. Fig. 7.22 . Fig. 7.23 composed of superficial
tissue strips ofwhich
around the glands, endometrial epithelium
is a clue to the artifactual resulting in a signet-ring
and cervical tissue in appearance,
endometrial maybiopsy occur secondary
specimens.
Scanty endometrial biopsy. Endometrial pipelle biopsy in Telescoping (glands within glands). This is a commonwith a pseudopapillary
nature architecture to The
thermal artifact
cervical tissue usually takes the form of strips of
postmenopausal woman where the specimen consists artifact in endometrial biopsy specimens
entirely of superficial strips of atrophic endometrial glands
without accompanying endometrial stroma
although theoretically endometrial type glands with or
without stroma could be derived from tuboendometrial
lower uterine segment. Paradoxically, it often takes the metaplasia or endometriosis within the cervix.
pathologist longer to examine such specimens since no It has been suggested that with an endometrial biopsy
underlying architecture is present, and the tissue must be containing scant tissue that cannot be typed, the term
examined carefully under high power to look for mitotic unassessable is more appropriate than inadequate or
activity, which is abnormal in a postmenopausal endome- insufficient [120]. In such cases, the gynecologist should
trium. In specimens such as this, it is controversial as to correlate the biopsy results with the ultrasonic and/or
what constitutes an adequate or inadequate specimen. hysteroscopic findings. If there is a clinical suspicion of
Designation of a biopsy as inadequate may be of impor- hyperplasia or malignancy, for example, if there is recur-
tance since this can have management and medicolegal rent postmenopausal bleeding, or if the ultrasonic and/or
implications. For example, some clinicians routinely per- hysteroscopic findings are worrying, then D and C should
form a repeat biopsy when an earlier sample has be performed. If the above investigations suggest an . Fig. 7.25 . Fig. 7.26
been reported as inadequate while others do not. atrophic endometrium, rebiopsy is probably unnecessary. Pseudopapillary endometrium. Endometrial biopsy Vacuolation of endometrial stromal cells. Vacuolation,
A biopsy reported as inadequate may suggest to some composed of superficial strips of endometrial epithelium resulting in a signet-ring appearance, may occur secondary
that the clinician is at fault or has not undertaken the with a pseudopapillary architecture to thermal artifact
biopsy procedure correctly. While this may be the case in Artifacts in Endometrial Biopsy
some instances, in most it is not. In published studies, Specimens
inadequate rates of outpatient endometrial biopsies range
from 4.8 to 33% [1, 2, 22, 52, 94], but in most of these There are several common artifacts in endometrial biopsy
studies, the criteria for adequacy are not clear. It is also specimens that have received scant attention in the litera-
menstruation and lymphocytes, including lymphoid individu
aggregates, are a normal component of the endometrial endomet
Besid
of lymph
promine
occasion
Other in
of endo

Endometritis
Usually,
mixed w
large nu
foamy cy
this is re
(> Fig.
occur in
or carci
obstruct
In en
. Fig. 7.31
involve t
Squamou
Endometritis is a histological diagnosis based upon an abnormal pattern of Acute endometritis. In acute endometritis, neutrophils are
present, sometimes forming microabscesses within
thelial m
nuclear
inflammatory infiltrate; as such, it must be distinguished from the normal glandular lumina

hematopoietic component of the endometrium

Most cases of endometritis occur in the reproductive years but sometimes


postmenopausal women are affected. Presentation is typically with abnormal vaginal
bleeding, most commonly intermenstrual bleeding or menorrhagia. .

Predisposing factors to endometritis include a recent pregnancy, the presence of an


intrauterine device (IUD), cervical stenosis, and prior instrumentation. Endometritis . Fig. 7.32 . Fig. 7.
may also accompany a pathological lesion within the uterus, such as an endometrial Chronic endometritis. In cases of chronic endometritis,
plasma cells are present within the endometrial
Xanthog
foamy hi
polyp, hyperplasia, carcinoma, or a leiomyoma. stroma

Benign Diseases of the Endometrium 7


stroma

32

Endometritis has traditionally been divided into acute and chronic forms, but these
constitute a continuum, and often there is an admixture of acute and chronic
inflammatory cells.

Endometritis may be focal or diffuse and can range from a subtle finding to a
pronounced inflammatory reaction. Usually, the endometrial glands exhibit
proliferative activity, and there may be mild glandular architectural distortion, in the
form of occasional dilated glands. There is often associated surface breakdown with
features identical to those seen in menstrual breakdown and breakdown due to non-.Syndecan
Fig. 7.34
immunohistochemical stain in endometritis.
. Fig. 7.35
CD20 in chronic endometritis. Chronic endometritis
menstrual causes. Syndecan (CD138) immunohistochemical staining may be
useful in identifying plasma cells. The endometrial glands
in which increased numbers of B lymphoid
cells (CD20 positive) are present within the
are also positive endometrial stroma

In some cases, an initial low-power clue to the diagnosis of endometritis is spindlenucleoli. As stated, sometimes there are mild architectural Immunohistochemical staining with B lymphoid markers
cell alteration of the stroma although this feature is not specific and is not alwayschanges with occasional dilated glands, but significant
glandular crowding is not a feature of endometritis.
(CD20 and CD79a) may also assist in distinguishing
between the physiological endometrial lymphocytic infil-
present. In other cases, the stroma may be edematous. As stated, there may be problems in identifying trate and the inflammatory infiltrate of endometritis.
plasma cells when they are few in number, especially in Normally, the vast majority of lymphoid cells within
suboptimally stained sections. Endometrial stromal cells the endometrial stroma are T cells (CD3 positive) with
may have a plasmacytoid appearance, especially pre- B lymphocytes accounting for about 1% of all endometrial
decidualized cells in the mid-and late-secretory phase, leucocytes [96]. B lymphoid cells are largely confined to
and unequivocal plasma cells with eccentric nuclei and lymphoid aggregates within the endometrial basalis with
a perinuclear hof should be present. Occasional plasma occasional individual cells in the functionalis. In most
positive filamentous bacteria may be found in the gyne-
cological tract. Because of the potential complications,

7
actinomyces must be distinguished from pseudoacti-
nomycotic radiate granules (pseudo-sulfur granules). 328 Benign Diseases of the Endometrium Benign Diseases of the Endometrium

lymphocytes, plasma cells, neutrophils, and histiocytes. acid-Schiff (PAS) stains are helpful in identifyin
Lymphoma-like lesions represent an exaggerated form of organisms.
chronic endometritis. The polymorphic nature of the Schistosoma, Enterobius vermicularis, and Echin
infiltrate together with the presence of germinal centers granulosus are rare causes of endometritis in de
and the superficial location of the inflammation (as stated, countries but schistosomiasis is endemic in som
only appreciated on hysterectomy or endometrial resec- of the world. Schistosomal endometritis may be
tion specimens) help to distinguish lymphoma-like lesion severe and is characterized by granulomatous infl
from malignant lymphoma, as does the absence of a mass tion with lymphocytes, plasma cells, eosinophils, a
lesion grossly. Immunohistochemistry for kappa and tiocytes, sometimes closely simulating a tuberc
lambda light chains or molecular investigations to dem- endometrial surface may be ulcerated and repla
onstrate a polyclonal population may also be of value. granulation tissue. Diagnosis is made by identify
Occasional cervical cases have been associated with ova in tissue sections or in smears of vaginal sec
Epstein–Barr virus infection [158]. Toxoplasmosis (Toxoplasma gondii) evokes a non
. Fig. 7.38
inflammatory reaction in the endometrium. The
Granulomatous endometritis. A single granuloma is present
organism can be identified by immunofluorescenc
. Fig. 7.37
Endometrial Granulomas within the endometrial stroma
Pseudoactinomycotic radiate granules.
Pseudoactinomycotic
Granulomas withinradiate granules witharethick
the endometrium rare. World- Malakoplakia
. Fig. 7.36 irregular club-like
wide, peripheral
the most common projections
cause is without a central
tuberculosis and,
material and a surrounding histiocytic and giant cell reac-
tion, the features resembling a rheumatoid nodule, may
Actinomycotic granules. Actinomycotic granules (AMGs) although rare in developed countries, granulomatous
dense core Malakoplakia may involve several organs, most com
occur secondary to endometrial ablation; usually, the
composed of thin basophilic radiating filaments with endometritis should be considered as tuberculous in ori- the urinary bladder, and is characterized by the pre
entire endometrium or much of the endometrium is
gin until proven otherwise. Tuberculosis of the endome- sheetsandof well-circumscribed
foamy histiocytesgranulomas
(von Hansemann’s
a dense more eosinophilic granular core affected, are not gen-hist
trium usually occurs in premenopausal women and is rare containing Michaelis–Gutmann bodies. These
These are erally found. A similar picture may be seen secondary to ar
afternoninfectious
the menopause. lesions, most commonly
Caseous necrosis seenofin
is characteristic round, laminated calcospherites, which are presen
association with granulomas
an IUD, but butdue
sometimes in non-IUD endometrial resection. In rare instances, there is no obvi-
tuberculous to the constant shedding
users. They consist of thick irregular club-like peripheral ouscytoplasm
cause forof endometrial
the histiocytes and in an extracellul
granulomatous inflam-
associated with menstruation, endometrial granulomas tion. They contain calcium and can be demon
projections without > Fig. 7.37). mation, so-called idiopathic granulomatous endometritis
in patients witha tuberculosis
dense central core (noncaseating.
are often
An associated inflammatory response may be present. (>byFig.von
7.38).Kossa stain. The histiocytes are often a
Tubercle bacilli are seldom identified on Ziehl–Neelson with other inflammatory cells, including plasm
With Brown and Brenn
stained sections, stains,should
and culture therebeisundertaken
diffuse, intense
in all and neutrophils. Occasional examples have been r
nonspecific
cases instaining, while examination
which histological silver stainsraisesare negative.
the possibil- Ligneous (Pseudomembranous)
in the endometrium [145]. Malakoplakia is a r
Pseudoactinomycotic
ity of tuberculosis.radiate granulescauses
Other infectious are probably more
of granuloma- Endometritis
an abnormal immune response to bacteria, mos
common tousthan actinomyces
endometritis [110]
include andfungi,
various occasionally the two
schistosomiasis,
monly Escherichia coli, which are retained wit
7
7
Benign Diseases of the Endometrium 353

of
me- to one third of uterine serous carcinomas, a lower fre- flattened stromal cells with occasional histiocytes and/or
ed quency than in ovarian serous carcinomas [58]. More giant cells. Spontaneous resolution usually occurs. Benign Diseases of the Endometrium 351
e is
uncommonly, they are seen in other morphological types Emphysematous endometritis should be distinguished
be
re- of endometrial malignancy, such as endometrioid carci- from sectioning artifact, dilated vascular spaces, and gas
es, noma. Psammoma bodies are occasionally seen in normal gangrene of the uterus, which is life threatening and asso-
n- endometria (usually atrophic or proliferative in type), ciated with tissue necrosis.
fil- sometimes in association with hormonal preparations,
of
and in benign lesions, most commonly endometrial
m-
polyps [59, 146]. They are often located within glandular
Benign Endometrial Stromal
of
en lumina ( Fig. 7.77), in which case it is likely that they
>
Proliferations
bly represent calcification of inspissated secretions. In other
but . Fig.
Endometrial stromal neoplasms are discussed in
cases, they7.72
are situated within the stroma where they may
on; > Chap. 10, Mesenchymal Tumors of the Uterus, as is
Mirena coil–associated endometrium. There is a low-power
ng be secondary to prior inflammation or the presence of an
polypoid architecture the differential diagnosis of fragments of tissue composed
on IUD. In the absence of a malignant lesion, the occurrence
ith
entirely of endometrial stroma in an endometrial biopsy.
of psammoma bodies within glandular lumina or endo-
s is Occasional cases of multifocal microscopic benign endo-
metrial stroma is not an indication for evaluation of the
on metrial stromal proliferations confined to the endome-
ces upper female genital tract to exclude malignancy. How-
trium without invasive growth have been reported [138].
ever, free-floating psammoma bodies without attachment
These have been termed focal endometrial stromal hyper-
ne to tissue may be an indication of an extrauterine serous
at plasia and may mimic an endometrial stromal nodule or
carcinoma.
in endometrial stromal sarcoma in biopsy samples. Rarely,
nd markedly atypical stromal cells with a symplastic appear-
ith ance are present within an otherwise normal endome-
ary Emphysematous Endometritis trium [148] (> Fig. 7.78).
ur.
There have been occasional reports of emphysematous
(pneumopolycystic) endometritis characterized by the Benign Trophoblastic Lesions
presence of gas-filled cysts in the.endometrial
Fig. 7.74stroma . Fig. 7.75
ng Mirena
[118, 149]. There may be simultaneous coil–associated
involvement of endometrium.
Benign lesions Stromal
of intermediate hyaline
trophoblast, namely pla- Radiation effect endometrium. Glands are lined by cells
ra- theFig. or the condition may be confined to the endo- cental site nodule or plaque (PSNP) and exaggerated pla-
cervix 7.73
.
ge-
metrium. Histology
Mirena
nodules
shows empty cystic
coil–associated spaces may
of be seen
variable
endometrium. centalin Mirena
site,
There
coil–associated
are discussed in > Chap. 20, Gestational
is stromal
with enlarged atypical nuclei

7
to
n-
expansion andthe
size and contour within
present within the stroma
endometrium
decidualization.
endometrial stroma lined byHemosiderin
Trophoblastic Tumors pigment is
and Related Tumor-Like Lesions.
en 354 Benign Diseases of the Endometrium
ent
ng
decidualization or pseudodecidualization with infiltration
so- by granulated lymphocytes (> Fig. 7.73) [119]. Other development of uterine malignancies. These may be of
oid
ect
nodules have also been described ( Fig. 7.74) [56]. There
histological features found in some cases include>
myxoid or mucinous change, hemosiderin pigment and
stromal
any morphological type but carcinosarcomas are propor-
ro- may be associated progestational effects in the cervix,
glandular metaplastic changes. Stromal necrosis, infarc- tionally overrepresented.
um tion, and microcalcifications are found in a small percent-
lly including microglandular hyperplasia and stromal
age of cases. In some cases, plasma cells are present within
ak the stroma, indicating a coexistent chronic endometritis,
nd decidualization.
secondary to the presence of the IUD. Stromal hyaline

Effects of Endometrial Ablation or


Resection
Radiation Effects on the Endometrium
. Fig. 7.79 . Fig. 7.80
Endometrial ablation is commonly
Placental site nodule or plaque. A well-circumscribed lesion
undertaken as
Intravascular menstrual endometrium. Menstrual
. Fig. 7.77 .
The endometrial morphology can be altered by the effects
Fig. 7.78 a nonsurgical procedure in the management
is composed of epithelioid cells with abundant cytoplasm
endometrium within of abnormal
myometrial blood vessels is
Psammoma bodies in endometrium. Psammoma Atypical endometrial stromal cells. Rarely endometrial occasionally seen and is of no significance
ofanradiation,
bodies within glandular lumina within endometrial which may have been administered many An endometrial
stromal cells with atypical symplastic-like nuclei are present uterine bleeding, especially in premenopausal women
PSNP may be identified in a biopsy, resec-
polyp years earlier. Radiation effect endometrium is character-
in the normal endometrium tion, where the suspicion
or hysterectomy specimen many of yearsmalignancy
following is low and preservation
a pregnancy or abortion and rarely even in a postmeno- important in all uteri but particularly so with endometrial
ized by surface endometrium or glands lined by cells with pausalofwoman. fertility is not an
PSNP is characterized issue.by The
histologically object
neoplasms of endometrial
where autolysis may result in problems in
a well-circumscribed lesion composed of cells with large assessing tumor type and grade. Bisection of the uterus
enlarged atypical, sometimes bizarre, hyperchromatic degenerate, ablation is destruction
often atypical nuclei and abundant of the soon
eosino- entire endometrium
after surgery may improve fixation,and
but this often
Adenomyosis
• A nonneoplastic lesion of myometrial tissue
characterized by the presence of endometrial glands
and stroma within myometrium Synonyms: myometrial
endometriosis, superficial adenomyosis (1 - 2.5 mm in
myometrium), stromal adenomyosis, incomplete
adenomyosis, adenomyosis with sparse glands
• Usually an incidental finding in hysterectomy
specimens
• May be diffuse or focal
• May be involved by hyperplasia and carcinoma
Terminology
• Adenomyoma: A circumscribed nodular aggregate of
benign endometrial glands surrounded by endometrial
stroma, with leiomyomatous smooth muscle bordering
the endometrial stromal component

Pathophysiology
• May result from (a) chronic uterine autotraumatization
by physiological mechanical functions and (b) tissue
injury and repair
• May be caused by disease of junctional zone
• Prolactin and immune factors may play a role
• Adenomyosis and endometriosis are usually regarded as closely
related, but
◦ Microscopic appearance, and probably their pathogenesis,
are somewhat different
◦ They may occur independently of each other
◦ Adenomyosis mostly is made up of nonfunctional (basal)
endometrium and is frequently connected with the mucosa
(vs. endometriosis, composed of functional layers)
◦ Adenomyosis may represents a unique form of endometrial
diverticulosis
• Hypothetical mechanisms include (Instillation of endometrium
within the myometrium
◦ In situ metaplasia of pluripotent stem cells retained in
myometrium or
◦ Improper partitioning of the endometrium from the
myometrium
• del(7) (q21.2q31.2), a deletion found in typical leiomyoma, has been
found in three cases of adenomyosis, suggesting some
pathobiologic overlap between leiomyomata and adenomyosis
Clinical features
• Nonneoplastic condition presenting with palpably enlarged uterus
• Symptoms are nonspecific: dysmenorrhea, menorrhagia, abnormal
uterine bleeding, dyspareunia, chronic pelvic pain associated with
the menstrual period and infertility (
• Associated with deep infiltrating endometriosis, parity, intense
dysmenorrhea and increasing age
• Tends to regress after menopause .
• When extensive, it confers a potential risk of infarction and
WHO 2014 Endometrial Hyperplasia Diagnostic Schema
EIN Nomenclature Topography Functional Treatment
category

Endometrial hyperplasia Diffuse Estrogen effect Hormonal therapy


without atypia(Unopposed
oestrogen,disordered
proliferation)
Endometrioid intraepithelial Focal Precancer Hormonal or
neoplasia(EIN) atypical progressing to Surgical
endometrial hyperplasia
diffuse
Glands>stroma,cytology,
size>1mm exclude carcinoma)
Carcinoma Focal Cancer Surgical stage
progressing to based
diffuse
for carcinoma {1019,1566}. Risk-factor logical atypia that exceeds that of normal thelial neoplasia (EIN).
associations include obesity, polycystic proliferative endometrium but falls short
ovarian syndrome and diabetes {504}. of the crowding seen in hyperplasia has ICD-O code 8380/2
Endometrial Hyperplasia Clinical features
been termed “disordered proliferative
phase”. Synonyms
Hyperplasia without atypia results from Complex atypical endometrial hyperpla-
without atypia prolonged oestrogen exposure unop- Histogenesis
posed by progesterone or progestational Hyperplasia without atypia is the result of
sia; simple atypical endometrial hyper-
plasia; endometrial intraepithelial neo-
Endometrial hyperplasia without atypia is an agents. It is most commonly diagnosed unopposed oestrogenic stimulation. plasia, EIN
exaggerated proliferation of glands of irregular size and in the perimenopause, with symptoms of
abnormal, non-cyclical vaginal bleeding.
shape, with an associated increase in the gland to
stroma ratio compared with proliferative endometrium,
but without significant cytological atypia.
Also called Benign endometrial hyperplasia; simple
non-atypical endometrial hyperplasia; complex non-
atypical endometrial hyperplasia; simple endometrial
hyperplasia without atypia; complex endometrial
hyperplasia without atypia
Rates of endometrial hyperplasia with- out atypia are
several-fold higher than for carcinoma
Risk-factor associations include obesity, polycystic
ovarian syndrome and diabetes A B
Fig. 5.01 Hyperplasia, without atypia. A Architectural changes include glandular branching, dilatation and crowding. B Cells lining the glands are columnar with cigar-shaped nuclei
Clinical features and are perpendicular to the basement membrane.
Hyperplasia without atypia results from prolonged
oestrogen exposure unopposed by progesterone or Epithelial tumours and precursors 125
progestational agents. It is most commonly diagnosed
in the perimenopause, with symptoms of abnormal,
non-cyclical vaginal bleeding.
Endometrial Hyperplasia
without atypia
A B
Fig. 5.01 Hyperplasia, without atypia. A Architectural changes include glandular branching, dilatation and crowding. B Cells lining the glands are columnar with cigar-shaped nuclei
and are perpendicular to the basement membrane.
Macroscopy Epithelial tumours and precursors 125
The endometrium varies from the uniform, 5 mm thick, tan appearance of late proliferative
phase to highly thickened, sometimes polypoid or spongy with cysts.
Histopathology
A spectrum of changes is typical. Glands vary in size and shape and may be sepa- rated by
varying amounts of stroma in cluding back-to-back crowding with little intervening stroma.
As this lesion is due to unopposed oestrogenic stimulation, the duration and dose of oestrogen
exposure affects the overall appearance .
Glands are irregularly distrib- uted, creating a variable density of glands to stroma. While some
glands may have normal coiled architecture, others branch or are cystically dilated. The
epithelium is of stratified columnar type, with frequent mitotic figures.
Focal haemorrhage and stromal breakdown are common. Proliferation of glands displaying no
cytological atypia that exceeds that of normal proliferative endometrium but falls short of the
crowding seen in hyperplasia has been termed “disordered proliferative phase”.
Atypical hyperplasia / Endometrioid intraepithelial neoplasia

Definition
Cytological atypia superimposed on endometrial
hyperplasia defines atypical hyperplasia (AH) /
endometrioid intraepi thelial neoplasia (EIN).
Synonyms
Complex atypical endometrial hyperplasia; simple
atypical endometrial hyper plasia; endometrial
intraepithelial neoplasia, EIN
Epidemiology
The average patient age at presentation is 53 years
Endogenous or exogenous hyperoestrinism is a risk factor A B
Fig. 5.02 Atypical hyperplasia/endometrioid intraepithelial neoplasia. A Architectural changes including aggregates of glands that exceed the volume of stroma. Glandular crowding
Clinical features is visible at low magnification. B The cytology of the affected glands (right and left mid field) differs from that of background glands and includes nuclear enlargement, rounding, loss
of polarity, pleomorphism and prominent nucleoli.
Postmenopausal bleeding or abnormal vaginal bleeding
Epidemiology these features are somewhat subjec- Prognosis and predictive factors
in perimenopausal women is the most common The average patient age at presentation tive, intraobserver and interobserver One-quarter to one-third of women with
presenting symptom. AH/EIN coexists with carcinoma in is 53 years {1588,1732}. Endogenous or variability remains problematic. AH/EIN a biopsy of AH/EIN will be diagnosed
approximately 25–40% of women exogenous hyperoestrinism is a risk fac- is often accompanied by metaplastic with cancer at immediate hysterecto-
tor {1917}. changes which have no bearing on clini- my or during the first year of follow-up
cal outcome, but as they display nuclear {81,998,1323,1931}. Longer-term risk
Clinical features rounding and enlargement, metaplastic elevation estimates vary from 14-fold in
Postmenopausal bleeding or abnormal changes add to the difficulty in diag- classic, early studies of AH {998} to 45-
vaginal bleeding in perimenopausal nosing nuclear atypia. Accordingly, the fold in EIN studies {81}.
women is the most common presenting diagnosis of atypia is facilitated by com-
symptom. AH/EIN coexists with carcino- parison of non-metaplastic epithelium to
Atypical hyperplasia / Endometrioid intraepithelial neoplasia
Macroscopy
The endometrium may be diffusely thickened up to 1 cm and may present as a visible
focal thickening resembling a polyp.
Histopathology
AH/EIN is composed of crowded aggregates of cytologically altered tubular or
branching glands. Within the geographic confines of the lesion, the area of glands
exceeds that of stroma, resulting in glandular crowding with little intervening stroma.
The distinction between endometrial hyperplasia without atypia and AH/EIN is based
on nuclear atypia which may include enlargement, pleomorphism, rounding, loss of
polarity and nucleoli. Nuclear atypia is variable, both qualitatively and quantitatively.
Histogenesis
Continuous unopposed oestrogenic stimulation leads to progression of hyperplasia
A B
without atypia to AH/EIN. Fig. 5.02 Atypical hyperplasia/endometrioid intraepithelial neoplasia. A Architectural changes including aggregates of glands that exceed the volume of stroma. Glandular crowding
is visible at low magnification. B The cytology of the affected glands (right and left mid field) dif ers from that of background glands and includes nuclear enlargement, rounding, loss
Genetic profile of polarity, pleomorphism and prominent nucleoli.
AH/EIN contains many of the genetic changes seen in endometrioid endometri- al these features are somewhat subjec- Prognosis and predictive factors
Epidemiology
carcinoma {1180}. These include micro- satellite instability, PAX2 inactivation and The average patient age at presentation tive, intraobserver and interobserver One-quarter to one-third of women with
PTEN, KRAS, and CTNNB1 (︎-catenin) mutation is 53 years {1588,1732}. Endogenous or variability remains problematic. AH/EIN a biopsy of AH/EIN wil be diagnosed
exogenous hyperoestrinism is a risk fac- is often accompanied by metaplastic with cancer at immediate hysterecto-
Genetic susceptibility tor {1917}. changes which have no bearing on clini- my or during the first year of fol ow-up
cal outcome, but as they display nuclear {81,998,1323,1931}. Longer-term risk
Hereditary susceptibility for AH/EIN parallels that of heritable syndromes associated Clinical features rounding and enlargement, metaplastic elevation estimates vary from 14-fold in
Postmenopausal bleeding or abnormal changes add to the difficulty in diag- classic, early studies of AH {998} to 45-
with an increased risk for endometrioid endometrial carcinoma. vaginal bleeding in perimenopausal nosing nuclear atypia. Accordingly, the fold in EIN studies {81}.
women is the most common presenting diagnosis of atypia is facilitated by com-
Cowden syndrome and Lynch syndrome (hereditary nonpolyposis colon cancer) symptom. AH/EIN coexists with carcino- parison of non-metaplastic epithelium to
ma in approximately 25–40% of women adjoining normal glands when present, Endometrial carcinomas
been termed “disordered proliferative
Clinical features phase”. Synonyms
Hyperplasia without atypia results from Complex atypical endometrial hyperpla-
WHO 2014 Endometrial Hyperplasia Diagnostic
prolonged oestrogen exposure unop- Histogenesis sia; simple atypical endometrial hyper-
posed by progesterone or progestational Hyperplasia without atypia is the result of plasia; endometrial intraepithelial neo-
agents. It is most commonly diagnosed
in the perimenopause, with symptoms of
Schema unopposed oestrogenic stimulation. plasia, EIN

abnormal, non-cyclical vaginal bleeding.

A B
A B Fig. 5.02 Atypical hyperplasia/endometrioid intraepithelial neoplasia. A Architectural changes including aggregates of glands that exceed the volume of stroma. Glandular crowding
is visible at low magnification. B The cytology of the affected glands (right and left mid field) differs from that of background glands and includes nuclear enlargement, rounding, loss
Fig. 5.01 Hyperplasia, without atypia. A Architectural changes include glandular branching, dilatation and crowding. B Cells lining the glands are columnar with cigar-shaped nuclei of polarity, pleomorphism and prominent nucleoli.
and are perpendicular to the basement membrane.
Epidemiology these features are somewhat subjec- Prognosis and predictive factors
The average patient age at presentation tive, intraobserver and interobserver One-quarter to one-third of women with
Epithelial tumours and precursors 125
is 53 years {1588,1732}. Endogenous or variability remains problematic. AH/EIN a biopsy of AH/EIN will be diagnosed
exogenous hyperoestrinism is a risk fac- is often accompanied by metaplastic with cancer at immediate hysterecto-
tor {1917}. changes which have no bearing on clini- my or during the first year of follow-up
cal outcome, but as they display nuclear {81,998,1323,1931}. Longer-term risk
Clinical features rounding and enlargement, metaplastic elevation estimates vary from 14-fold in
Postmenopausal bleeding or abnormal changes add to the difficulty in diag- classic, early studies of AH {998} to 45-
vaginal bleeding in perimenopausal nosing nuclear atypia. Accordingly, the fold in EIN studies {81}.
women is the most common presenting diagnosis of atypia is facilitated by com-
symptom. AH/EIN coexists with carcino- parison of non-metaplastic epithelium to
Complex Atypical Hyperplasia “CAH
Gland crowding (Gland : Stroma ratio usually >2:1), often
densely crowded with only small amounts of intervening
stroma and back-to-back glands
Nuclear Atypia: relatively enlarged, rounded nuclei with loss
of polarization, chromatin abnormalities (often clearing or
vesicular) and variably prominent nucleoli
Often important to compare nuclei to non-hyperplastic glands
elsewhere in specimen to account for fixation artifact, etc..
If nuclear atypia is too much, consider other diagnoses like
endometroid adenocarcinoma, serous carcinoma, etc

Although CAH and EIN are often


overlapping diagnoses, they
have slightly different diagnostic
criteria. EIN is likely more
specific, while CAH is more
sensitive.
Endometrial Intraepithelial Neoplasia (EIN) Both are acceptable by the
Conceived as a true carcinoma precursor based on WHO and usage often depends
architectural and genetic abnormalities interpreted on local practice.
as neoplastic.
Diagnostic criteria:
1- Crowded glands with a gland to stroma ratio >1:1
2- Altered cytology of the crowded glands from the
background epithelium c l gic de a ca i
This can be nuclear or cytoplasmic.
- If no background epithelium is present, use same
criteria as for CAH (above)
3- Must be > 1mm within a single tissue fragment

EIN/CAH: Nuclei are enlarged Proliferative Endometrium: Cigar-


and rounded with chromatin like dense nuclei with basal, often
clearing and loss of polarity pseudostratified, arrangement


WHO Classification of tumours of the uterine corpusa,b
Epithelial tumours and precursors Dissecting (cotyledonoid) leiomyoma 8890/0
Precursors Diffuse leiomyomatosis 8890/1
Hyperplasia without atypia Intravenous leiomyomatosis 8890/1
Atypical hyperplasia / Endometrioid Metastasizing leiomyoma 8898/1
intraepithelial neoplasia 8380/2* Smooth muscle tumour of uncertain malignant
Endometrial carcinomas potential 8897/1
Endometrioid carcinoma 8380/3 Leiomyosarcoma 8890/3
Squamous differentiation 8570/3 Epithelioid leiomyosarcoma 8891/3
Villoglandular 8263/3 Myxoid leiomyosarcoma 8896/3
Secretory 8382/3 Endometrial stromal and related tumours
Mucinous carcinoma 8480/3 Endometrial stromal nodule 8930/0
Serous endometrial intraepithelial carcinoma 8441/2* Low-grade endometrial stromal sarcoma 8931/3
Serous carcinoma 8441/3 High-grade endometrial stromal sarcoma 8930/3
Clear cell carcinoma 8310/3 Undifferentiated uterine sarcoma 8805/3
CHAPTER 5 Neuroendocrine tumours Uterine tumour resembling ovarian sex cord
Low-grade neuroendocrine tumour tumour 8590/1*
Tumours of the uterine corpus Carcinoid tumour 8240/3 Miscellaneous mesenchymal tumours
High-grade neuroendocrine carcinoma Rhabdomyosarcoma 8900/3
Epithelial tumours and precursors Small cell neuroendocrine carcinoma 8041/3 Perivascular epithelioid cell tumour
Large cell neuroendocrine carcinoma 8013/3 Benign 8714/0*
Mesenchymal tumours
Mixed cell adenocarcinoma 8323/3 Malignant 8714/3*
Undifferentiated carcinoma 8020/3 Others
Mixed epithelial and mesenchymal tumours
Dedifferentiated carcinoma
Miscellaneous tumours Mixed epithelial and mesenchymal tumours
Tumour-like lesions Adenomyoma 8932/0
Lymphoid and myeloid tumours Polyp Atypical polypoid adenomyoma 8932/0
Metaplasias Adenofibroma 9013/0
Secondary tumours Arias-Stella reaction Adenosarcoma 8933/3
Lymphoma-like lesion Carcinosarcoma 8980/3

Mesenchymal tumours Miscellaneous tumours


Leiomyoma 8890/0 Adenomatoid tumour 9054/0
Cellular leiomyoma 8892/0 Neuroectodermal tumours
Leiomyoma with bizarre nuclei 8893/0 Germ cell tumours
Mitotically active leiomyoma 8890/0
Hydropic leiomyoma 8890/0 Lymphoid and myeloid tumours
Apoplectic leiomyoma 8890/0 Lymphomas
Lipomatous leiomyoma (lipoleiomyoma) 8890/0 Myeloid neoplasms
Epithelioid leiomyoma 8891/0
Myxoid leiomyoma 8896/0* Secondary tumours
a
The morphology codes are from the International Classification of Diseases for Oncology (ICD-O) {575A}. Behaviour is coded /0 for benign
tumours, /1 for unspecified, borderline or uncertain behaviour, /2 for carcinoma in situ and grade III intraepithelial neoplasia and /3 for
malignant tumours; b The classification is modified from the previous WHO classification of tumours {1906A}, taking into account changes in
our understanding of these lesions; *These new codes were approved by the IARC/WHO Committee for ICD-O in 2013.
TNM and FIGO classification of carcinomas of the
uterine endometrium
T – Primary tumour N – Regional Lymph Nodes

TNM FIGO NX Regional lymph nodes cannot be assessed


TX Primary tumour cannot be assessed N0 No regional lymph node metastasis
T0 No evidence of primary tumour N1 Regional lymph node metastasis
Tis Carcinoma in situ (preinvasive carcinoma)
T1 Ia Tumour confined to the corpus uteria M – Distant Metastasis
a
T1a IA Tumour limited to endometrium or invading less
than half of myometrium M0 No distant metastasis
T1b IB Tumour invades one half or more of myometrium M1 Distant metastasis (excluding metastasis to vagina,
T2 II Tumour invades cervical stroma, but does not ex- pelvic serosa, or adnexae, including metastasis to
tend beyond the uterus inguinal lymph nodes other than para-aortic or pelvic
T3 and III Local and/or regional spread as specified below: nodes)
/or N1
T3a IIIA Tumour invades the serosa of the corpus uteri or Stage grouping
adnexae (direct extension ormetastasis)
T3b IIIB Vaginal or parametrial involvement (direct exten- Stage IA T1a N0 M0
sion or metastasis) Stage IB T1b N0 M0
N1 IIIC Metastasis to pelvic or para-aortic lymph nodesb Stage II T2 N0 M0
IIIC1 Metastasis to pelvic lymph nodes Stage IIIA T3a N0 M0
N2 IIIC2 Metastasis to para-aortic lymph nodes with or with- Stage IIIB T3b N0 M0
out metastasis to pelvic lymph nodes Stage IIIC T1, T2, T3 N1, N2 M0
T4 IVA Tumour invades bladder/bowel mucosac Stage IIIC1 T1, T2, T3 N1 M0
M1 IVB Distant metastasis (excludes metastasis to vagina, Stage IIIC2 T1, T2, T3 N2 M0
pelvic serosa or adnexae) Stage IVA T4 Any N M0
Stage IVB Any T Any N M1
a
Notes: Endocervical glandular involvement only should now be considered
b
as Stage I. Positive cytology has to be reported separately without chang-
c
ing the stage. The presence of bullous oedema is not sufficient evidence to
classify as T4. This lesion should be confirmed by biopsy.

References
American Joint Committee on Cancer (AJCC) Cancer Staging Manual, 7th ed. (2011) Edge SB, Byrd DR, Compton CC, Fritz AG, Greene FL, Trotti III eds.
Springer: New York
International Union against Cancer (UICC): TNM Classification of Malignant Tumours, 7th ed. (2009) Sobin LH, Gospodarowicz MK, Wittekind Ch eds.
Wiley-Blackwell: Oxford
A help-desk for specific questions about the TNM classification is available at http://www.uicc.org.
Endometrial tumors(Neoplasms)
• Endometrial Glandular • Precursor to Serous
Neoplasia carcinoma
• Endometrioid Intraepithelial • Endometrial
Neoplasia-A precursor to adenocarcinomas
Endometrioid(Type1) • Endometrioid
Endometrioid adenocarcinoma
adenocarcinoma
• Serous,Clear cell carcinomas
• Tumors with mixed epithelial
and stromal differentiation
• Endometrial Stromal tumors
• Non mullerian Neoplasms
Classification and Grading of Endometrial
Carcinoma
• Endometrial • Endometrial adenocarcinoma
adenocarcinoma Non endometrioid
Endometrioid type(Type11) grade 3 by
type(Type1) grades 1-3 definition
• Serous
• With Secretory
Differentiation • Clear cell
• Carcinosarcoma
• Squamous
Differentiation • Endometrial adenocarcinoma
Miscellaneous(Squamous
• Mucinous Differentiation cell,Mixed,Undifferentiated)
• Rare-Transitional types
An Approach to Difficult Endometrial Lesions Modified from: Mills and Longacre. Surg
Pathol Clin. 2011 March

High-Grade Cytology?
No Yes

Architecture Adenocarcinoma

Be sure to consider:
Serous
Low-risk Intermediate-risk High-risk Clear cell

Note: If the proliferation has


CAH/EIN a papillary or labyrinthine
<30% <30% growth pattern, there is no %
CAH/EIN Borderline Carcinoma
Borderline requirement.

Architectural Patterns:
High-risk Patterns: Carcinoma
- Glandular confluence without intervening
stroma (10x field)
- Extreme, meandering or labyrinth pattern
- Macroglands with well-developed secondary
branching or multiple generations of bridging
forming a cribriform pattern
- Villous and nonvillous papillae containing
second and third degree branching and/ or
cribriform budding

Intermediate-risk Patterns: Borderline

Low-risk Patterns: Hyperplasia


- Simple budding into small glands
- Macroglands with simple, non-branching
nonvillous papillae or minimal bridging
- Simple non-branching villous and nonvillous
papillae
Endometrial carcinoma – risk factors
Tamoxifen and endometrial carcinoma risk
1. Hyper-estrogenic state (endogenous or exogenous) most important risk
factor for type I endometrioid endometrial adenocarcinoma (type II cancers • Tamoxifen is a nonsteroidal compound that competes with estrogen for
older thin built postmenopausal women) estrogen receptors
• Unopposed estrogen therapy (without progestin)
• In women of childbearing age it antagonizes endogenous estrogens
• Tamoxifen
endometrial inactivity or atrophy
• Early menarche and late menopause
• Obesity • In postmenopausal women, who are normally hypo-estrogenic, it may have
• Diabetes a weak estrogenic effect.
• Polycystic ovary syndrome
• Tamoxifen administration is associated with 2 to 3 times increased risk of
• Ovarian sex-cord stromal tumors (e.g. granulosa cell tumor) endometrial adenocarcinoma, usually early stage and low grade

2. Heritable risk hereditary non-polyposis colorectal carcinoma syndrome • A small subset of patients have aggressive high-grade endometrioid
(HNPCC) or Lynch syndrome 70% lifetime risk of endometrial carcinomas, clear cell carcinomas, or carcinosarcomas
adenocarcinoma tend to develop 15 years earlier than sporadic cancers
and the prognosis is favorable • Women treated with tamoxifen are also particularly prone to developing
endometrial polyps, often gigantic size
Cigarette smoking reduces the risk of endometrial carcinoma
An Oversimplified View of Endometrial Adenocarcinoma:
Types 1 and 2
. Table 9.3 bladder a
Architectural grading of endometrial carcinoma bowel mu

Endometrial Carcinoma 9 Grade


401 1 No more than 5% of the
IVB G123 Distant
metastase
tumor is composed of solid
The most recent revision of the FIGO (International including
Federation of Gynecology and Obstetrics) Staging System masses
(> Table 9.4) and the WHO Histopathologic Classifica-
intraabdo
tion of uterine carcinoma recommend that tumors Grade 2 6–50% of the tumor is and/or in
be graded using both architectural and nuclear criteria composed of solid masses lymph no
[89, 356]. The grade of tumors that are architecturally
grade 1 or 2 should be increased by one grade in the Grade 3 More than 50% of the tumor
presence of ‘‘notable’’ nuclear atypia, defined as grade 3 G1, 5% or less of a nonsquamous or nonmorular solid grow
nuclei [438]. For example, a tumor that is grade 2 by
404 9 Endometrial Carcinoma
Endometrial Carcinoma
9
is composed of solid masses
403
G2, 6–50% of a nonsquamous or nonmorular solid growth p
more than 50% of a nonsquamous or nonmorular solid grow
which stands for microcystic, elongated, and fragmented Rules on staging:
1. Corpus cancer is now surgically staged. Those patients w
[279]. In the study cited, MELF invasion was associated undergo a surgical procedure should be staged according
with lymphovascular invasion, but not with poor progno- FIGO clinical staging.
sis in a multivariate analysis. 2. Ideally, the thickness of the myometrium should be meas
It may be difficult to distinguish myometrial invasion with the depth of tumor invasion.
from the extension of the carcinoma into adenomyosis Notes on grading:
1. Notable nuclear atypia, inappropriate for the architect
(> Fig. 9.19). The distinction, however, is important raises a grade 1 or grade 2 tumor by one grade.
because the presence of carcinoma in adenomyosis deeper 2. In serous adenocarcinomas, clear cell adenocarcinomas
. Fig. 9.2 . Fig. 9.8 than the maximum
. Fig. 9.10 depth of true tumor invasion does not mous cell carcinomas, nuclear grading takes precedence.
EndometrioidEndometrioid carcinoma,
grade 1FIGO grade 1 (architectural Endometrioid carcinoma, FIGO grade 2 (architectural
carcinoma, FIGO
grade
grade 1, nuclear 1, nuclear
grade 1). Well-formed
(architectural
grade 1). Nuclei
glandsare round
have
worsen
to oval
small, with the
gradeprognosis
2, nuclear grade [164,
. Fig.
1).9.12 176,
Well-formed glands195, 271]. When the . Fig.3.9.14Adenocarcinomas with squamous differentiation are grad
are admixed

and grade ing tocarcinoma,


the nuclear
uniform chromatin nonsquamous nests of tumor, with the latter 2 (architectural
Endometrioid carcinoma, FIGO grade Endometrioid FIGO grade of the glandular component.
grade 3 (architectural
round to oval nuclei with uniform chromatin carcinomawith issolidsurrounded
grade
5%2,but
bygrade
nuclear endometrial
2). Tumor stroma
is composed of intimately a3, nuclear grade 3). Solid nonsquamous tumor with
comprising more than less than 50% of the overall Endocervical gland involvement should be considered sta
residual benign
tumor glands are present in these foci, the diag- foci
admixed glandular and solid nonsquamous epithelium
within an edematous and inflamed altered stroma
of necrosis and rare residual glandular lumens displays
b
notablePositive peritoneal
nuclear atypia fluid
characterized cytology should be reported sep
by nuclear
nosis of carcinoma extending into adenomyosis is enlargement does and notpleomorphism
affect the withstage.
vesicular chromatin
and prominent nucleoli
straightforward. At times, however, the distinction
from myometrial invasion may be extremely difficult,
particularly in older women in whom adenomyosis may

. Fig. 9.3
Endometrioid.carcinoma,
Fig. 9.9 FIGO grade 1 (architectural . Fig. 9.11
Endometrioid
grade 1, nuclear carcinoma,epithelium
grade 1). Glandular FIGO gradedisplays
1 (architectural
a Endometrioid carcinoma, FIGO grade 2 (architectural
gradeindicating
confluent pattern 1, nuclear endometrial
grade 2). Nuclei are somewhat
stromal invasion enlarged, grade 2, nuclear grade 1). Glandular and solid areas have
by carcinoma
rounded, and have granular to vesicular chromatin with . Fig. round
generally uniform small, 9.13 to oval nuclei with granular . Fig. 9.15
occasional small nucleoli chromatin Endometrioid carcinoma, FIGO grade 3 (architectural Endometrioid carcinoma, FIGO grade 3 (architectural
Grade 1 EEC Grade 2 EEC

Grade 3 EEC High-grade nuclear atypia in this tumor with no


solid growth (grade 1 architecture) increase the
grade by one unit to grade 2
Novel molecular classification of endometrial carcinomas
• The Cancer Genome Atlas (TCGA) classification of endometrial carcinomas
somatic copy number alterations (SCNAs) and tumor mutation
burden (TMB) four distinct EC classes
1. POLE - mutant EC recurrent mutations in the exonuclease domain of
DNA poly-merase epsilon (POLE) gene involved in nuclear DNA
replication and repair these ECs are copy number stable but have high
somatic mutation frequencies (>100 mutations per megabase)

2. Microsatellite instability (MSI) associated EC hypermutated due to


defect in MMR genes (MLH1, PMS2, MSH2, and MSH6) and
microsatellite instability high mutation frequencies (> 10 but < 100
mutations per Mb) but copy number stable

3. EC with PI3K/Akt and Wnt signalling pathways mutation Genomically


relatively stable, MMR-proficient, with moderate number of mutations
mostly within the PI3K/Akt and Wnt signalling pathways

4. EC with TP53 mutation high somatic copy number alteration (SCNA)


POLE mutant ECs
• Grade 3 endometrioid EC with ambiguous morphology
• Superficial broad front invasion pattern, presence of tumor giant cells and
prominent tumor-infiltrating lymphocytes
• High somatic mutation frequencies (>100 mutations per Mb)
• Somatic copy number alteration is very low
• Low BMI patients, early stage and excellent prognosis
• Diagnosis: NGS/ Sanger sequencing
MSI associated ECs
• Grade 3 Endometrioid EC with tumor infiltrating lymphocytes and peri-
tumoral lymphocytes, more common in LUS (Non-endometrioid subtypes
can also occur )
• A so-called ‘microcystic elongated and fragmented (MELF)’ pattern of
invasion and LVSI have also been associated.

• Sporadic (epigenetic silencing of MLH1) or syndromic (Lynch)


• Endometrial carcinoma develops before colon carcinoma in >50% of women
with HNPCC
• Defect in MMR genes (MLH1, PMS2, MSH2, and MSH6) MSI high
mutation frequencies (> 10 but < 100 mutations per Mb)
• Somatic copy number alteration is low

• Higher BMI, intermediate prognosis


• Diagnosis: MMR protein IHC / MSI PCR assay
ECs with PI3K/Akt and Wnt signalling pathways mutation

• Grade 1-2 endometrioid ECs with ER and PR positivity


• Squamous differentiation is frequent

• Most common gene mutated is PTEN / CTNNB


• < 10 mutations/ Mb
• Somatic copy number alteration is low

• Higher BMI, variable prognosis (excellent / intermediate / poor)


TP53 aberrant/mutant ECs

• High-grade (grade 3) endometrioid ECs and serous carcinoma


• Destructive invasion and frequent LVSI

• < 10 mutations/ Mb but somatic copy number alteration is very


high
• p53 mutation and IHC interpretation gain of function mutation
(IHC: diffuse strong nuclear +) or loss of function missense mutation
(IHC: loss of p53 staining or cytoplasmic staining) All or none
phenomenon (p53 wild type: IHC shows patchy staining)
• Mutation of p53 can also happen in a region/subclone of tumor
instead of the whole tumor

• Lower BMI, late onset advanced stage, poor prognosis


The molecular classification presented by the TCGA was built upon EC with
endometrioid and serous morphology, leaving the question of whether it can be
translated to other rare EC subtypes
Algorithm for molecular classification of ECs
Therapeutic implications for
different molecular subtypes
of endometrial carcinoma in
addition to pan-hysterectomy
Helpful Tables
IHC in common subtypes of endometrial carcinoma Carcinoma Immunohistochemistry
Endometrioid Serous Clear Cell
(Low-grade)
ER/PR + -/+ -/+
p53 Wild-type Abnormal Wild-type, usually
P16 -/patchy Block-positive -/patchy
PTEN Loss Intact Intact
NapsinA - -/+ +
HNF1 - -/+ +
All three subtypes are CK7+, CK20-, PAX8+ However, as always, there are exceptions. For example, grade 3 endometrioid carcinomas may exhibit
(this phenotype is useful in evaluation of metastatic carcinoma from unknown primary) a sero s imm nophenot pe ith p m tations ia dedifferentiation and rare clear cell carcinomas
may also stain with p53.

Endometrial Polyp Classification and Treatment


Diagnosis Epithelium Stroma Management
Benign Endometrial Polyp Benign Benign, Fibrous Polypectomy
Polypoid adenomyoma Benign Benign, Muscle Polypectomy
Endocervical vs. Endometrial adenocarcinoma: IHC
Grossing
Procedure (tumor)
• Measure, weigh, describe, photograph
• Open along lateral sides
• Ink serosa adjacent to tumor, vaginal margin of resection
• Section from endocervical canal to superior aspect
• If cervical tumor present / suspected, amputate cervix and process as
cone biopsy
Sections to obtain
No significant gross pathology:
• Anterior and posterior cervix (important for identifying carcinoma in situ,
• Anterior and posterior endomyometrium
• Lower uterine segment
• Each ovary (cortex, hilar region) and fallopian tube
• Any gross lesions, including leiomyomas

Suspected tumor:
• Above, plus:
• With tumor, full thickness endomyometrium and serosa to assess depth
of invasion
• 1 section per 1 cm of tumor, minimum 3 sections
• All grossly different appearing regions of tumor
• High endocervical canal or lower isthmus
• Ink and obtain margins of resection, including vaginal mucosa

• Notes: in laparoscopic hysterectomies, pathologists may create


artifactual vascular invasion by mechanically transporting tumor into
vascular spaces during the grossing procedure
• May be difficult to identify specific cervical anatomic regions in
morcellated specimen
Endometrial carcinoma – core data elements
• Specimen Integrity
• Tumor Site
• Tumor Size (changed from required to optional)
• Histologic Type
• Histologic grade
• Myometrial Invasion most important prognostic determinant
• Tumor Involvement of Cervix
• Peritoneal wash / Ascitic Fluid
• Lymph-Vascular Invasion
• Regional Lymph Nodes
• Distant Metastasis (changed to required only if confirmed pathologically)
• Additional Pathologic Findings – e.g. hyperplasia, EIN
• Biomarker – ER & PR, MMR protein IHC, p53 IHC
GROSS DESCRIPTION HISTOLOGY
Specimen:
_________________________________________________________ ____________ differentiated _____________
________ Uterus with cervix measures ____ X ____ X ____ cm.

An ulceroproliferative / ____________ tumour is seen in the fundus /
adenocarcinoma of the Endometrium. (FIGO grade ).
body / isthmus/ _______ ____________________________ of the The tumour is limited to the endometrium.

uterus measuring ____ X ____ X ____ cm. The tumour invades less than inner half / equal to or
The tumour invades less than / more than half the thickness of the more than half of the myometrium. Uterine serosa is not
myometrium.
 involved / involved.

Uterine serosa is
Distance of tumour from the uterine serosa is _____ cm.

_________________________________________________________
____ Isthmus is involved / not involved.
 Lymphatic emboli are seen / not seen.

The cervix is involved not involved
 Perineurial invasion is seen/ not seen.

Myometrium shows The cervix is free of tumour.

_________________________________________________________ The tumour involves the cervical glands and / or stroma.

__ Vaginal cuff is absent / present and measures ___________ The vagina is free / involved _____The vaginal cut
The vagina is involved / not involved. margins are ____
The right ovary measures ____ X ____ X ____ cm and is The right and left parametria are ____________The
______________________________ The right fallopian tube is _____ adjacent endometrium shows __
cm long and is ______________________________________ The left
ovary measures ____ X ____ X ____ cm and is The myometrium ____
_______________________________ The left fallopian tube is _____
cm long and is _______________________________________ Right The right ovary is ______
parametrium measures ____ X ____ X ____ cm and is
___________________________. Left parametrium measures ____ X The right fallopian tube is _
____ X ____ cm and is ____________________________. The left ovary is ___________The left fallopian tube is
Lymph nodes: ______________
Sections: The omentum _____________
Lymph nodes:
Myometrial invasion
• Depth of invasion should be measured from the endomyometrial junction
(EMJ) to the deepest point of invasion
• EMJ in normal conditions is often irregular look for compressed, non-
neoplastic endometrial glands at the nearby EMJ or even at the base of the
tumor

• Carcinoma involving adenomyotic foci should not be interpreted as


invasive carcinoma this distinction at times may be difficult as some
invasive carcinoma do not elicit a stromal response
• In the rare cases where myoinvasive carcinoma is only encountered in foci of
adenomyosis involved by carcinoma distance from the adenomyotic focus
to the deepest area of invasion should be measured, otherwise over-staging
may be done (e.g. if there is a tumor with a 2-mm focus of myoinvasion from
a focus of adenomyosis in the deep myometrium, it is still considered as
having <50% myometrial invasion or FIGO stage IA).
Schematic of measurement of depth of invasion in (A) tumor with a regular interface; (B)
tumor with an irregular endomyometrial interface; (C) and (D) tumor with an exophytic
growth; (E) tumor arising from adenomyosis.
MELF type invasion appears restricted to low grade, myoinvasive carcinomas
of endometrioid type, more frequently in tumors exhibiting focal mucinous
differentiation and LVSI IHC for CK may be required to diagnose
AJCC- TNM and FIGO staging of endometrial carcinoma
Endometrioid Carcinoma
Most common carcinoma of the endometrium.
Classically, in post-menopausal women and related to
increased levels of estrogen exposure (associated with
obesity, diabetes, PCOS, and certain medications) and
preceded by hyperplasia.
Often presents with vaginal bleeding (always a concerning
finding after menopause!)
Crowded, complex glandular or villoglandular
architecture.
Cells are often columnar and share an apical border with
Cribriform growth
eosinophilic granular cytoplasm. Nuclear atypia is often
mild to moderate.
Distinction from CAH/EIN is based on stromal invasion,
which is defined by one of the following
1- Loss of intervening stroma: confluent growth,
cribriform growth, or complex folded mazelike epithelium
2- Irregular infiltration of myometrium associated with an
altered fibroblastic stroma (desmoplastic response)
3- Solid nonsquamous epithelial growth
4- Papillary architecture or villoglandular growth Villoglandular growth
Some cut- ff f he e e f c fl e g h be
more objective have been suggested:
Kurman & Norris = 2 mm
Longacre = 30% of total proliferation and should be able
to traverse a 10x field without hitting stroma
Sometimes the cutoff of carcinoma vs CAH cutoff can be
challe gi g OK diag ea CAH b de i g on well-
differentiated endometrioid ade ca ci a Squamous morules
Frequently see squamous differentiation with morules,
keratin pearls, and intercellular bridges.
Occasionally see secretory changes with glycogen vacuole
or bland spindled epithelial cell component.
Corded and Hyalinized: has cords, clusters, and/or
trabeculae of epithelioid to spindled cells embedded within
hyalinized to myxoid matrix. Often associated with grade 1-
2 glandular component. No prognostic significance, but
sometimes confused with carcinosarcoma, leading to
overtreatment.
IHC: (+)CK7, PAX8; (-)CK20; Cup-like vimentin staining.
Low-grade often ER/PR +;
FIGO grade % Solid Growth
Grade using the FIGO system: based on the amount of solid
growth. Be sure to exclude squamous morules from this 1
calculation. The grade can be increased by 1 based on severe
2 6-50%
nuclear atypia in the majority of the cells. A confluent
microacinar pattern is often counted as solid. 3 >50%
Endometrioid Carcinoma (continued)
Molecular Classification
Ultramutated (POLE mutated) subtype (~5%): Mutations of DNA polymerase result in an extremely high
tumor mutation rate lots of neoantigens in tumor cells recognized by immune system lots of tumor
infiltrating lymphocytes Excellent prognosis. Often younger patients. Often grade 3 with intratumoral
heterogeneity and giant tumor cells. Broad invasive front with low clinical stage.

Hypermutated/Microsatellite Instability (MSI) subtype (~25%): Mutations in mismatch repair proteins


result in a high mutation rate lots of neoantigens in tumor cells recognized by immune system lots
of tumor infiltrating lymphocytes. Intermediate prognosis despite TILs. Often Grade 3, substantial LVI,
MELF-pattern of invasion. Often located in lower uterine segment. Associated with Lynch Syndrome.

Copy Number High/Serous-like subtype (~25%): Genomically unstable high somatic copy number
alterations. Very high rate of TP53 mutations. Often Grade 3 with diffuse high nuclear grade, slit-like
spaces, hobnailing, and destructive invasion. Often older patients and advanced stage. Poor prognosis.

Copy Number Low/Microsatellite Stable (MSS) subtype (~45%): Most common type. Often Grade 1-2,
ER/PR+ with squamous differentiation. Associated with unopposed estrogen exposure (as is seen in
obesity). Overall low mutation rate. Very frequent PTEN mutations. Intermediate prognosis (depends
largely on stage).

Generally, frequent mutations (can be seen all groups) in PTEN, PIK3CA, ARID1A, CTNNB1, and KRAS

In general:
Low-grade (FIGO 1/2): map to the copy number low and MSI-H categories
High-grade (FIGO 3): map to all 4 categories, but least to the copy number low group

Familial syndromes:
Lynch Syndrome: germline mutations in mismatch repair (MMR) proteins MSI-subtype
~50% lifetime risk (similar to risk of colon cancer).
PTEN-hamartoma tumor syndrome/Cowden syndrome: Germline PTEN mutation no specific morphology

Molecular Classification Algorithm:


Endometrioid
POLE mutation Adenocarcinoma
POLE wild type
Ultramutated/
POLE-mutated MMR loss MMR intact

Hypermutated/MMR-deficient p53 overexpressed or null


p53 wild type

Copy number low


P53-mutated/ Serous-like

How do you separate serous carcinoma from serous-like endometrioid carcinoma?


Primarily by morphology (e.g., if there is squamous differentiation serous-like endometrioid).
However, both are TP53-mutated and are aggressive, and this can be morphologically challenging,
so this distinction is likely not too important at this time.
Myoinvasion
Endometrium
Can be very challenging to identify/measure! Tumor

Depth of Invasion
Measure from endomyometrial junction to the deepest point of

Myometrial Thickness
invasion.

Report as % (depth of invasion/total myometrial thickness).

Critical clinical cutoff point = 50% (inner vs outer ½)

Myometrial invasion often (but not always!) includes irregular angular


Myometrium
glands eliciting a desmoplastic response or loose granulation tissue
with inflamed edematous fibrous stroma

It can be helpful to look for compressed non-neoplastic glands to


determine the level of the endomyometrial junction.

Carcinoma involving adenomyosis or within vessels do not count as


invasion. So, look for surrounding benign glands and stroma to rule
out colonization of adenomyosis

Challenging patterns of myometrial invasion:


P hi g I a i broad, expansile front with a mild or absent
stromal reaction. Helpful to submit adjacent normal endometrium to
determine level of endomyometrial junction (if present).

Mic c ic E ga ed a d F ag e ed MELF invasive glands


often lined by a single, flattened layer of epithelium with eosinophilic,
squamoid cytoplasm. Simulate vascular spaces. Associated
edematous, inflamed stroma. Very sneaky!! More aggressive.

Diff e i fi a i e melter a e individual well-formed glands


with mild to moderate atypia that diffusely infiltrate the myometrium
i h minimal mal eac i n f like aden ma malign m f
the cervix). Often wide-spread throughout uterus. Make Dx on low-
power from architecture. Can be extremely challenging to evaluate
for superficial invasion, but good prognosis if stage 1, so less
important.

Lymphovascular Invasion
Frequently seen with MELF pattern.
S me ime in a a c la cell can a ea hi i c id
requiring stains to confirm that they are tumor.
Can ee f e en a c la e d in a i n i h
laparoscopic hysterectomy specimens, which is thought to
be artifactual/iatrogenic. So, if it is a low-grade, non-
in a i e m ha a em ed la a c icall i
babl e d in a i n
Other Endometrial Carcinomas
Serous Carcinoma
Epithelial cells with large atypical nuclei, prominent nucleoli, and
scant cytoplasm. Numerous mitoses.
Often complex papillary architecture. Can be solid or glandular.
Luminal surfaces often appear scalloped (no common apical
border as is seen in endometrioid). Grading not applicable.
Of en infil ra e in gaping o li -like (non-solid) glands
Typically post-menopausal women presenting with bleeding.
Often grossly inconspicuous on the surface of a polyp.
Background endometrium often atrophic.
Serous Endometrial Intraepithelial Carcinoma SEIC non-
invasive precursor to serous carcinoma; confined to the
epithelium (e.g., surface of a polyp). Malignant: Can still undergo
transtubal metastasis to pelvis.
Molecular: Frequent TP53 mutations. Associated with BRCA1/2.
IHC: p53 mutant (either diffuse or null), P16 block positive.
Prognosis depends on stage (advanced = very bad).

Clear Cell Carcinoma


Polygonal or hobnail-shaped cells with clear cytoplasm (or
sometimes eosinophilic) and prominent nuclear atypia
Tubulocystic, papillary, or solid architecture with hyalinized
stroma and eosinophilic extracellular hyaline globules.
Often atrophic background.
Often postmenopausal women with vaginal bleeding.
Grading not applicable.
Relatively poor prognosis.

Mucinous Carcinoma
An endometrial carcinoma in which > 50% of the neoplasm is mucinous. Very rare.
(Most tumors that are mucinous are endometrioid adenocarcinoma with mucinous differentiation)
Often low-grade with glandular or villoglandular architecture and uniform mucinous columnar cells
with minimal stratification/atypia. Frequent KRAS mutations. Can still grade using FIGO System.
Relatively good prognosis.
Be sure to consider endocervical origin on biopsy!

Mixed Carcinoma
The term mixed carcinoma should be used when two or more distinctive subtypes of endometrial
carcinoma are identified, each representing at least 5% of the tumor.
chromatic nuclei. It should be noted that considerable
changes (cribriform and papillary patterns) induced by
nuclear heterogeneity can be observed.
progestin treatment, are noteworthy as they may be con-
Villoglandular carcinomas are generally better differ-
fused with progression [424]. Importantly, biopsies taken
entiated than typical endometrioid carcinomas but are not
after the initiation of treatment require a comparison
significantly different with respect to depth of invasion or
to the pretreatment sample for correct interpretation
frequency of nodal metastases [442]. In addition,
and determination of treatment response (see > Chap. 8,
villoglandular carcinomas are frequently admixed with
Precursor Lesions of Endometrial Carcinoma). It has
typical endometrioid carcinoma. In view of the frequent
been shown that patients with complex atypical hyperpla-
admixture of the two patterns and similar prognosis,
sia and well differentiated carcinoma who were success-
villoglandular carcinoma is considered a variant of endo-
fully treated with progestins had higher numbers of NK
metrioid carcinoma. Treatment is the same as for en-
cells and lower numbers of regulatory T cells in post
dometrioid carcinoma of comparable stage, grade, and
treatment specimens. These alterations in subpopulations
depth of invasion.
of lymphocytes suggest that the effect of progestins goes
. Fig.their
beyond 9.24direct growth regulatory effects on endome-
. Fig. 9.32 . Fig. 9.29 . Fig. 9.31carcinoma, secretory type, FIGO grade 1. . Fig. 9.22
trialEndometrioid
tissue [426].
Secretory
Serous carcinoma. Papillary tumor is lined by markedly Mucinous Carcinoma
carcinoma, FIGO grade 1. Confluent glands have Endometrioid-type
Mucinous carcinoma, FIGO grade 1 (endometrioid Endometrioid carcinoma, villoglandular type. Tumor has
epithelium displays prominent sub- and
atypical epithelium composed of cells with scalloped abundant mucinous cytoplasm and small, basally situated supranuclear
carcinomavacuolization,
with mucinous papillary architecture, which might lead to misclassification
reminiscent of day 18 secretory a
differentiation). Tumor exhibits
luminal borders, including hobnail type cells nuclei carcinoma is a variant of typical endometrial Villoglandular
Secretory Carcinoma
cribriform growth
endometrium pattern and is comprised of glands with as serous carcinoma, but columnar epithelium with low-
carcinoma in which the majority of cells exhibit subnu- prominent mucinous cytoplasm as well as ones with a grade cytologic features (see > Fig. 9.23) is consistent with
clear or supranuclear cytoplasmic vacuoles resembling Villoglandular carcinoma is
typical endometrioid a variant of endometrioid
appearance endometrioid carcinoma
early secretory endometrium. An unusual pattern, it rep- carcinoma that displays a papillary architecture in which
resents only 1–2% of endometrial carcinomas [229, 402]. the papillary by typical fronds are composed
endometrioid of a delicate
carcinoma with lessfibrovas-
than 50% of a
The age range is from 35 to 79 years, with a mean age of cular core coveredcomponent
mucinous by columnar can cells
be that generally
designated as contain
endometrioid
55–58 [75, 402]. Most patients are postmenopausal and bland nuclei carcinomas[69, 174].
with The mediandifferentiation
mucinous age is 61 years, (>similar
Fig. 9.31).
experience abnormal bleeding. This histologic subtype to that of women with typical endometrioid carcinoma. In
also may be seen after progestin treatment of an all other respects, women with these tumors are similar to
endometrioid carcinoma. In all other respects, including patients Differential
with low-grade Diagnosis
endometrioid carcinoma.
the association of obesity, hypertension, diabetes mellitus, The microscopic appearance of villoglandular carci-
and exogenous estrogen administration, patients with noma is Endocervical
characterized epithelium merges with
by thin, delicate frondsthecovered
endometriumby in
secretory carcinoma are similar to women with stratified the columnar
lower uterine segment,
epithelial cellssowith
it is not
ovalsurprising
nuclei that that the
endometrioid carcinoma. distinction
generally display mild of primary
to moderate endocervical
(grade 1 or from endometrial
2) atypia
Microscopically, secretory carcinoma displays a well- (> Figs. mucinous
9.22 andcarcinoma in curettings can be difficult. There
> 9.23). Occasionally, more atypical

differentiated glandular pattern and is composed of (grade is3)no histochemical


nuclei difference
may be observed. in theactivity
Mitotic mucin is at var-
either site
columnar cells, often unstratified, with subnuclear or iable, and [336]. The distinction
abnormal mitotic of endocervical
figures are rarefrom [69, endometrial
174].
supranuclear vacuolization closely resembling day 17–22 Myometrial adenocarcinomas
invasion usually has been discussed earlier (see section
is superficial.
. Fig. 9.30 . > Differential Diagnosis for endometrioid carcinoma).

9
>
secretory endometrium ( Figs. 9.24–9.26) [229, 402]. Fig. 9.25
Mucinous carcinoma, FIGO grade 1. Tumor has extensive Endometrioid The distinction
carcinoma,ofsecretory
mucinous carcinoma
type, FIGO grade of the
1. endo-
Usually the nuclei are grade 1. The secretory pattern may . Fig. 9.23
Endometrial Carcinoma mucinous differentiation and confluent glandular and Differential metriumDiagnosis
from clear cellwith
or secretory
secretorycarcinoma is madeare on
be focal or diffuse, and it is frequently admixed with Endometrioid-type glands differentiation
Endometrioid carcinoma, villoglandular type. Endometrioid
papillary growth; these architectural patterns allow for the
present basis
within ofamorphology
desmoplastic and PAS
stroma. and mucin
Prominent stains. The
endometrioid adenocarcinoma. The endometrium adjacent differentiation is confirmed by the presence of columnar
establishing a diagnosis of carcinoma Thesubnuclear
main
cells consideration
in in the differential diagnosis
vacuolization is reminiscent of day 17 secretory or
secretory carcinoma are clear (not is
granular
to secretory carcinoma in young women typically shows epithelium and low-grade cytologic features (elongated
serous carcinoma because both villoglandular
foamy) because of the presence of glycogen, which is PAS and serous
a secretory pattern that is more advanced than 17 days, endometrium uniform nuclei)
was present in a polyp in 27% of the cases [267]. carcinomas
The positive haveand a prominent
is removed papillary
by diastase pattern. In con-
treatment. Mucin in
and a corpus luteum is found in most premenopausal
presence of intracytoplasmic mucin can be identified trast
on tothese
seroustumors
carcinomas,
is villoglandular
focal at most. carcinomas
Clear cell have
carcinoma is
patients when a hysterectomy and bilateral salpingo- Differential Diagnosis
hematoxylinareand eosin (H&E) stains by long delicate
its distinctive almostpapillary fronds and
always papillary are covered
or solid by columnar
in contrast to the glandu- important distinguishing feature is the cytologic appear-
oophorectomy performed. Nonetheless, a relationship
granular, foamy, or bubbly appearance and can cells
be with
lar only mild
pattern of to moderate
mucinous nuclear atypia.
carcinoma. The Thein
cells cells
clear ance. The cells of serous carcinoma tend to be rounder,
cell
to progesterone stimulation is not always demonstrable. It is important to distinguish secretory carcinoma from clear
look distinctly endometrioid with arather smooth,
than luminal forming small papillary clusters that are detached from the
confirmed
In fact, secretoryby carcinoma
PAS, mucicarmine,
may occurorspontaneously in cell carcinoma
alcian blue stains. carcinomatend to beofpolygonal
in view the excellent columnar
prognosis of theand
border.hobnail
To have significance as invariably
a distinctive entity,a the papillary fronds, a finding that is often referred to as
The intracytoplasmic
postmenopausal mucin isexogenous
women without variable inor both the distri-
abnormal former and cells are almost
unfavorable prognosis of thepresent, cytologic
latter. Although
bution of mucinous cells in the tumor and in the location diagnosis
of is reserved
feature that is for tumors
absent in in which
mucinous most of the papillary tufts. As a consequence, the luminal border has
carcinoma.
levels of progesterone. The secretory activity in the tumor both tumors are composed of cells with clear, glycogen-rich
neoplasm Rarely,
has a villoglandular
a mucinous appearance.orIna contrast
carcinoma mixed to a scalloped appearance. The nuclei of serous carcinomas
mucinous
maythebe mucin within
transient individual
because cells.observed
it has been Mucin may be diffusely
in curettings cytoplasm, the histologic features are distinctive. At times a
villoglandular carcinomas,carcinoma
serous carcinomas tendareas
to have are highly pleomorphic and atypical (grade 3). Cherry
present in the cytoplasm, confined
but not in the later hysterectomy specimen [75]. to the apical area, or secretory carcinoma that has a predominantly glandularsim-
and endometrioid may contain that
show a combination of both patterns. Tumors dominated shorter,ulate
thick, densely fibrotic
microglandular papillary of
hyperplasia fronds. The most
the cervix [436, 445].red macronucleoli typically are present and the cells

. Fig. 9.27 . Fig. 9.28


Endometrioid carcinoma with squamous differentiation, Endometrioid carcinoma with focal squamous
FIGO grade 1. Well-differentiated glands are intimately differentiation, FIGO grade 3. Carcinoma has areas of
admixed with solid nests of low-grade squamous squamous differentiation and is classified as high-grade
receptors. The distinctionEndometrial
of serous Carcinoma 9
carcinoma from
clear cell carcinoma is discussed later (see > Clear Cell
419

Carcinoma).
At times papillary syncytial eosinophilic change, par-
ticularly in a small curettage specimen in an older patient,
may be difficult to distinguish from serous carcinoma. The
papillary processes in eosinophilic change lack fibrovas-
cular support and the cells that form these processes are
small and lack significant nuclear atypia or mitotic activ-
ity. Typically, small microcystic spaces containing neutro-
phils are present in the syncytial masses (see > Chap. 8,
Precursor Lesions of Endometrial Carcinoma). At times it
may not be clear if a serous carcinoma involving the endo-
metrium is primary or metastatic from the ovary. More
. Fig. 9.43
often than not the uterus is the primary site, even when
Serous carcinoma. Tumor exhibits diffuse/strong nuclear
invasion cannot be demonstrated in the hysterectomy spec-
. Fig. 9.37 . Fig. 9.39 expression of p53 which correlates with a p53 mutation
imen [420]. In these cases the ovarian involvement is
Serous carcinoma. Papillae are lined by cells with enlarged, Serous carcinoma. Papillae are lined by markedly atypical
vesicular nuclei with evident nucleoli. Several mitotic
typically bilateral and characterized by small foci of tumor
epithelium
figures are present
on the ovarian surface or nodules of tumor in the paren-
chyma with clusters of tumor cells in hilar vascular spaces.

Immunohistochemical Findings

Approximately 75% of endometrial serous carcinoma


show p53 overexpression (intense expression in greater
than 50–75% of tumor cell nuclei; > Fig. 9.43) as a result
of p53 mutation and the consequent accumulation of
mutant protein [234, 362, 381, 396, 448]. Some of the
remaining tumors, which show absolutely no p53 expres-
sion, have p53 mutations that result in a truncated p53
protein or a protein with conformational changes that
cannot be detected using commercially available anti-
bodies [396]. The Ki-67 labeling index is extremely high
(i.e., greater than 50–75% of tumor cell nuclei) [236]. The . Fig. 9.44
. Fig. 9.38 . Fig.
typical 9.40 carcinoma lacks diffuse ER and PR expres-
serous Serous carcinoma. Tumor exhibits diffuse/strong
Serous carcinoma. Glands with prominent intraglandular sionSerous carcinoma.
[59, 65, Some380],
100, 237, glandular epithelium
although many hascarcinomas
smooth expression of p16
papillary architecture infiltrate myometrium luminal borders but papillary epithelial tufts and
with hybrid endometrioid/serous features and admixtures marked
nuclear atypia, with
of endometrioid and numerous mitotic figures,
serous components are
express consid-
are frequently encountered. Mitotic activity usually is high characteristic of serous carcinoma and endometrioid carcinomas of the endometrium, these
erable amounts of ER [15]. PR is less frequently expressed,
and abnormal mitotic figures are easily identified.
compared to ER. Diffuse/strong p16 is characteristic of tumors share some notable features. Like endometrioid
Psammoma bodies are encountered in a third of cases. cases, the surface endometrium adjacent to the carcinoma
serous carcinomas (> Fig. 9.44) [73, 328, 433]. In contrast carcinomas, endometrial serous carcinomas commonly
The invasive component of the neoplasm can show con- or at other sites away from the neoplasm is replaced by one
to most endocervical carcinomas, this does not imply express pan-cytokeratins, EMA, CA125, Ber EP4, B72.3,
invasive, and to present
serous carcinoma, clear
quently associated with
nuclear grade, lymphova
lymph node metasta
endometrioid carcinom
confined to a polyp [
patients with clear cell c
reported in various ser
[2, 75, 229, 314, 421]. In
with tumor beyond stag
in stage I the 5-year surv
report of low-stage tumo
with an estimated surviv
. Fig. 9.51
. Fig. 9.47 . Fig. 9.49 nearly 97 patients the 5-
Clear cell carcinoma. Hobnail type cells have
Clear cell carcinoma. Glands are lined by cells with clear Clear cell carcinoma. Solid tumor is composed of cells with the 10-year survival, 31%
hyperchromatic nuclei and vacuolated clear cytoplasm
cytoplasm and many uniform cuboidal cells with focal prominent clear cytoplasm and characteristic nuclei with 10-year actuarial disea
hobnail features vesicular chromatin and nucleoli and 39% [7], and in an
CC was 29 months and 5
wide range in survival r
suggests that different in
ferent criteria for the d
resulting in a heterogene
Treatment is variable and
apy is used irrespective
radiation or chemothera
In view of the poor pro
often have high nuclear g
deeply, adjuvant therapy

Immunohistochemic
. Fig. 9.52
Clear cell carcinoma. Hyalinized papillae are lined by cells Like endometrioid and s
. Fig. 9.48 . Fig. 9.50
with clear to eosinophilic cytoplasm and pleomorphic cinomas usually express
Clear cell carcinoma. Papillary tumor exhibits characteristic Clear cell carcinoma. Hobnail type cells protrude
nuclei BerEP4, B72.3, CK7, and
features, including hyalinized stroma and cells with clear prominently into the gland lumen and are pleomorphic
to granular eosinophilic cytoplasm and hobnail with hyperchromatic nuclei negative for CK20 and W
morphology carcinomas. In some cases, mixtures of both types are plasmic expression of CE
found. Yolk sac tumors occur rarely in the endometrium Clear cell carcinom
abnormal mitoses are readily seen. PAS-positive, diastase- tumor. The differential diagnosis of the first tumor has but the patients are young, in contrast to women with and show p53, p16, and
resistant intracellular, and extracellular hyaline bodies, sim- been discussed (see > Secretory Carcinoma). Clear cell clear cell carcinoma, who are almost always post- mediate between endom
ilar to those in endodermal sinus tumors, can be found in carcinoma can be distinguished from serous carcinoma menopausal. Microscopically, yolk sac tumors often [237, 328, 413]. Notably,
nearly two-thirds of clear cell carcinomas. by architectural and cytoplasmic rather than nuclear fea- have a microcystic pattern that can resemble the ation indices that mimic
tures because both tumors display similar high-grade tubulocystic pattern of clear cell carcinoma. Character- not seen in clear cell carc
nuclear features, including vesicular nuclei with prominent istically, the yolk sac tumor contains Schiller–Duval carcinomas show abnor
Differential Diagnosis nucleoli, hobnail cells, and cells with hyperchromatic, bodies, which are lacking in clear cell carcinoma. Yolk and PTEN pathways [2
smudged nuclei. Serous carcinomas do not display the sac tumors are associated with elevated serum alpha- well-characterized cases
The differential diagnosis of clear cell carcinoma includes tubulocystic or solid growth patterns, clear cytoplasm, fetoprotein (AFP) levels, and AFP can be identified in carcinoma of the e
secretory carcinoma, serous carcinoma, and yolk sac and hyalinized stroma that are characteristic of clear cell the tumor by immunohistochemistry. a manifestation of HNP
Adenofibroma
Dense, fibrous connective tissue with
interspersed glandular spaces and
periglandular condensation of stromal cells.
No marked cellularity, nuclear atypia and
mitosis < 2 per 10 HPF.
Adenosarcoma

‘Phyllodes like architecture’


Periglandular stromal condensation with
nuclear atypia and mitosis >2 per 10 HPF in
stromal cells. Epithelium is benign.
IHC: Mesenchymal cells are CD10+, ER+,
PR+ , variable SMA +
\ Carcinosarcoma
Epithelial component glandular
architecture; may have an endometrioid,
clear-cell, serous, mucinous, yolk-sac
or mesonephric differentiation.
Mesenchymal component
homologous and heterologous elements

The epithelial and mesenchymal


components show concordant staining
for p53, supporting their common clonal
origin
Undifferentiated Carcinoma
Malignant epithelial neoplasm with no differentiation.
Sheets of medium-sized relatively uniform, monotonous,
discohesive cells with often condensed chromatin.
No gland formation (resembles lymphoma). Numerous
mitoses. Often numerous tumor-infiltrating lymphocytes (TIL)
Often form large, polypoid masses within uterine cavity.
Molecular: Often MMR-deficient (some Lynch-associated);
Frequent mutations of SWF/SNF pathway (SMARCA4,
SMARCA1/INI-1, ARID1B de-activation)
IHC: Loss of MLH1/PMS2. Loss/focal Cytokeratins, EMA, ER,
PAX8. (+)CD34
Very aggressive.

Dedifferentiated Endometrial Carcinoma


Two distinct components: 2
1) Low-grade (FIGO 1-2) endometrioid carcinoma 1
2) High-grade undifferentiated carcinoma (see above)

Each component has the IHC/molecular of that component


(e.g., undifferentiated component has SWF/SNF mutations).

Very aggressive.

Carcinosarcoma
Biphasic tumor with two components:
1
1) High-grade carcinoma (epithelial) and
2) Sarcoma (mesenchymal)
Typically post-menopausal women presenting with
vaginal bleeding. Often a large pelvic mass that
prolapses out of the cervix in ~1/2 of cases.
Often intimate admixture of carcinoma and sarcoma
elements.
2
Carcinoma is often serous or endometrioid carcinoma
Sarcoma is often high-grade non-specific sarcoma, but
he e ologo elemen can be seen including:
rhabdomyosarcoma, chondrosarcoma, and
osteosarcoma (which look/stain like they do elsewhere)
Molecular: Frequent TP53 mutations.
Poor prognosis. Frequent pelvic recurrences and lymph
node metastases (of carcinomatous component)
Rhabdomyoblastic differentiation
Questions
Question 1
Currently, staging of uterine sarcoma:

• Divides stage T3 / III disease depending on the number of abdominal sites involved by tumor
• Follows the same staging classification of uterine carcinoma
• Has a unified staging classification for pure mesenchymal and mixed epithelial mesenchymal tumors
• Incorporates lymphovascular space invasion as a staging criterion
• Incorporates tumor grade as a staging criterion
Answer 1
A. Divides stage T3 / III disease depending on the number of abdominal sites involved by tumor (IIIA: 1 site
involved; IIIB: > 1 site involved). The 8th edition of AJCC and FIGO (2015 and 2018 updates) have a separate
staging classification for uterine sarcomas, separate from uterine carcinoma staging. It contains two staging
systems: one for leiomyosarcoma and endometrial stromal sarcoma (and one can guess, other pure uterine
sarcomas) and a second one for adenosarcoma. Carcinosarcoma is staged as a uterine carcinoma. Tumor
grade and lymphovascular space invasion are not required for staging purposes.

Question 2
The defining feature of stage T3a / IIIA carcinoma of the uterine corpus is:

• Involvement of bladder or rectal mucosa


• Involvement of cervical stromal tissue
• Involvement of uterine serosa or adnexa
• Involvement of vagina or parametrial tissue
• Pelvic lymph node metastases
Answer 2
C. Involvement of uterine serosa or adnexa
Questions
Question 3

The carcinoma shown in the image above, which involves the surface of an endometrial polyp, shows
strong and diffuse positivity for p53 and p16. Which of the follow is true regarding the above carcinoma?

Likely arose in a patient less than 30 years of age


Considered an estrogen dependent endometrial carcinoma
Even without definitive invasion, can present at high stage with tumor dissemination in the peritoneal cavity
Often shows monotonous nuclei with little pleomorphism and low proliferation index
Answer 3
C. This is a serous endometrial intraepithelial carcinoma in an endometrial polyp. Even without definitive
invasion, can present at high stage with tumor dissemination in the peritoneal cavity.

Question 4
• Which of the following is true regarding serous carcinomas of the endometrium?

• Mutations in p53 are the most common driving molecular event


• WT-1 is often strong and diffusely positive
• Show well formed glands with sharp luminal borders
• p16 overexpression indicates a role for HPV infection in the etiology
Answer 4
A. Mutations in p53 are the most common driving molecular event.
Questions
Question 5
• A 32 year old woman with a history of pulmonary lymphangioleiomyomatosis, seizures and
angiofibromas presents with a myometrial mass. Which molecular / cytogenetic abnormality does this
patient likely have?

• HTR4-ST3GAL1 fusion
• RAD51B fusion
• TFE3 fusion
• TFEB fusion
• TSC2 mutation
Answer 5
E. TSC2 mutation

Question 6
• A 50 year old woman presents with a myometrial mass with lung metastases. The myometrial tumor
is characterized by dyshesive epithelioid and spindled cells surrounded by a delicate vasculature.
The tumor cells are strongly and diffusely positive for HMB45, desmin, SMA and caldesmon with focal
melanA expression. What type of tumor does this likely represent?

• Alveolar soft part sarcoma


• Leiomyosarcoma
• Melanoma
• PEComa
• Poorly differentiated endometrial carcinoma
Answer 6
D. PEComa
Questions
Question 8
Which of the following molecular alterations has been reported in
uterine leiomyosarcomas?

• ALK fusions
• BCOR fusions
• JAZF1 fusions
• TP53 mutations
• TSC2 mutations
Answer 8
D. TP53 mutations
Question 9
A uterine mass in a 50 year old woman has a tumor shown in the photomicrograph which expresses SMA, ER, PR
and caldesmon and is negative for CD10 and CD117 / KIT. What is your diagnosis?

• Endometrial stromal sarcoma


• Gastrointestinal stromal tumor
• High grade endometrial adenocarcinoma
• Leiomyosarcoma
Answer 9
D. Leiomyosarcoma

Question 10
A 44 year old woman presents with an infiltrative spindle cell neoplasm in the myometrium that has prominent
myxoid stroma and a dense lymphoplasmacytic infiltrate. It is positive for desmin, smooth muscle actin and ALK by
immunohistochemistry and shows ALK rearrangement by FISH. What is the most likely diagnosis?

• BCOR rearranged endometrial stromal sarcoma


• Inflammatory myofibroblastic tumor
• Myxoid endometrial stromal sarcoma
• Myxoid leiomyosarcoma
• Postoperative spindle cell nodule
Answer 10
B. Inflammatory myofibroblastic tumor
Questions
Question 13
Which of the following immunophenotypes is consistent with a well differentiated endometrial
endometrioid adenocarcinoma?

• PAX8+, CK7+, CK20+, ER / PR+, wild type p53, patchy / focal p16
• PAX8+, CK7+, CK20-, ER / PR+, wild type p53, strong / diffuse p16
• PAX8+, CK7+, CK20-, ER / PR-, wild type p53, strong / diffuse p16
• PAX8+, CK7+, CK20-, ER / PR+, wild type p53, patchy / focal p16
• PAX8+, CK7-, CK20+, ER / PR+, wild type p53, patchy / focal p16
Answer 13
D. PAX8+, CK7+, CK20-, ER / PR+, wild type p53, patchy / focal p16

Question 14
Which chromosomal abnormality is likely to be associated with the histologic findings from endometrial
biopsy of a 35 year old woman with abnormal uterine bleeding?

• 1p aberrations
• 6p21 aberrations
• 9q34 aberrations
• Loss of 7q
Answer 14
B. 6p21 aberrations. The histology is consistent with endometrial polyp. The most common cytologic
abnormality associated with endometrial polyps are aberrations of chromosome 6p21. Aberrations of
chromosome 1p, 9q34 and loss of 7q are associated with uterine leiomyoma.
References

R
THANK YOU

You might also like